Sei sulla pagina 1di 66

Total Marks : 200

Online Prelims TEST - 6 (SUBJECT WISE)


( InsightsIAS Mock Test Series for UPSC Preliminary Exam 2020 )

1 With reference to the Himalayan Rivers, consider the following statements:


1. They are perennial in nature receiving water from both glaciers and rain.
2. All the rivers are examples for consequent drainage pattern.
3. They are still in a youthful stage when compared to Peninsular rivers.

Which of the statements given above is/are correct?


A. 1 only
B. 1 and 3 only
C. 2 and 3 only
D. 1, 2 and 3

Correct Answer : B

Answer Justification :

Hence, statement 1 and statement 3 are correct whereas statement 2 is incorrect.

2 The tropic of cancer passes through which of the following states in India?
1. Gujarat
2. Rajasthan
3. Chhattisgarh
4. Bihar
5. Meghalaya

Select the correct answer using the code given below:


A. 1, 3 and 4 only
B. 1, 2 and 3 only
C. 3, 4 and 5 only
D. 1, 2 and 4 only

1
Total Marks : 200
Online Prelims TEST - 6 (SUBJECT WISE)
( InsightsIAS Mock Test Series for UPSC Preliminary Exam 2020 )

Correct Answer : B

Answer Justification :

Hence, option (b) is correct.

3 Consider the following passes of the Western Ghats:


1. Bhorghat
2. Thalghat
3. Palghat

Arrange the above given passes from South to North using the code given below:
A. 3-1-2
B. 3-2-1
C. 2-3-1
D. 2-1-3

Correct Answer : A

Answer Justification :

2
Total Marks : 200
Online Prelims TEST - 6 (SUBJECT WISE)
( InsightsIAS Mock Test Series for UPSC Preliminary Exam 2020 )

Bhor Ghat is a mountain passage located between Palasdari and Khandala for railway and
between Khopoli and Khandala on the road route in b, India, situated on the crest of the
Western Ghats.

Thal Ghat is a ghat section in the Western Ghats near the town of Kasara in Maharashtra. The Thal
Ghat is located on the busy Mumbai–Nashik route, and is one of the four major routes, rail and
road routes, leading into Mumbai.

Palakkad Gap or Palghat Gap is a low mountain pass in the Western Ghats between
Coimbatore in Tamil Nadu and Palakkad in Kerala. It has an average elevation of 140 metres
with a width of 24–30 kilometres. The pass is located between the Nilgiri Hills to the north and
Anaimalai Hills to the south.

4 With reference to Indian standard Time (IST) meridian (82°30’ E), consider the following statements:
1. It passes through eight Indian states.
2. It intersects Tropic of cancer in the state of Chhattisgarh.
3. Official time signals are generated by the Time and Frequency Standards Laboratory at the
National Physical Laboratory in New Delhi, for both commercial and official use.

Which of the statements given above is/are correct?


A. 1 only
B. 1 and 3 only

3
Total Marks : 200
Online Prelims TEST - 6 (SUBJECT WISE)
( InsightsIAS Mock Test Series for UPSC Preliminary Exam 2020 )

C. 2 and 3 only
D. 1, 2 and 3

Correct Answer : C

Answer Justification :

Standard Meridian of India which is 82°30′ East longitude passes through Uttar Pradesh,
Madhya Pradesh, Chattisgarh, Odisha and Andhra Pradesh. Hence, statement 1 is
incorrect.

Tropic of cancer and IST meridian intersect at Koriya district in Chhattisgarh. Hence,
statement 2 is correct.

Official time signals are generated by the Time and Frequency Standards Laboratory at
the National Physical Laboratory in New Delhi, for both commercial and official use. The
signals are based on atomic clocks and are synchronised with the worldwide system of clocks that
support the Coordinated Universal Time. Hence, statement 3 is correct.

5 Consider the following statements:


1. Alluvial soil is also known as ‘self-ploughing’ soil.
2. Black soil is important for growing of plantation crops.
3. Saline soils are found in arid and semi-arid regions, and in waterlogged and swampy areas.

Which of the statements given above is/are correct?


A. 1 and 3 only
B. 1 and 2 only
C. 3 only
D. 1, 2 and 3

Correct Answer : C

Answer Justification :

The black soils are generally clayey, deep and impermeable. They swell and become sticky
when wet and shrink when dried. So, during the dry season, these soil develop wide cracks. Thus,
there occurs a kind of ‘self ploughing’. Hence, statement 1 is incorrect.

Black soil is important for growing of crops like cotton and sugarcane. Laterite soil is more
suited to grow plantation crop. Hence, statement 2 is incorrect.

Saline soils are also known as Usara soils. Saline soils contain a larger proportion of sodium,
potassium and magnesium, and thus, they are infertile, and do not support any vegetative growth.
They have more salts, largely because of dry climate and poor drainage. They occur in arid and
semi-arid regions, and in waterlogged and swampy areas. Hence, statement 3 is correct.

4
Total Marks : 200
Online Prelims TEST - 6 (SUBJECT WISE)
( InsightsIAS Mock Test Series for UPSC Preliminary Exam 2020 )

6 Consider the following statements:


1. Western Ghats are more continuous when compared to the Eastern Ghats as the latter is eroded by
several east flowing rivers.
2. The Eastern and the Western Ghats meet each other at the Nilgiri hills.

Which of the statements given above is/are correct?


A. 1 only
B. 2 only
C. Both 1 and 2
D. Neither 1 nor 2

Correct Answer : C

Answer Justification :

Western Ghats are locally known by different names such as Sahyadri in Maharashtra, Nilgiri hills
in Karnataka and Tamil Nadu and Anaimalai hills and
Cardamom hills in Kerala. Western Ghats are comparatively higher in elevation and more
continuous than the Eastern Ghats.

Eastern Ghats comprising the discontinuous and low hills are highly eroded
by the rivers such as the Mahanadi, the Godavari, the Krishna, the Kaveri, etc. Some of the
important ranges include the Javadi hills, the Palconda range, the Nallamala hills, the Mahendragiri
hills, etc. The Eastern and the
Western Ghats meet each other at the Nilgiri hills. Hence, both statement 1 and statement
2 are correct.

7 Consider the following pairs:


Industry: State:
1. Korba Aluminum Plant Maharashtra
2. Rourkela Steel Plant West Bengal
3. Durgapur Steel Plant Chhattisgarh

Which of the pairs given above is/are correctly matched?


A. 1 and 2 only
B. 3 only
C. 1, 2 and 3
D. None

Correct Answer : D

Answer Justification :

BALCO has its operations at Korba in the state of Chhattisgarh with a smelter capacity of 570
ktpa with capabilities to produce ingots, Alloy ingots, wire-rods, busbars and rolled products.
Hence, pair 1 is incorrectly matched.
5
Total Marks : 200
Online Prelims TEST - 6 (SUBJECT WISE)
( InsightsIAS Mock Test Series for UPSC Preliminary Exam 2020 )

Rourkela Steel Plant, in Rourkela, Odisha is the first integrated steel plant in the public
sector in India. It was set up with West German collaboration with an installed capacity of 1
million tonnes in the 1960s. It is operated by Steel Authority of India. Hence, pair 2 is incorrectly
matched.

Durgapur Steel Plant is one of the integrated steel plants of Steel Authority of India Limited,
located in Durgapur, in the eastern Indian state of West Bengal. It was set up with the help of
United Kingdom. Hence, pair 3 is incorrectly matched.

8 Consider the following pairs:


Multipurpose project River
1.
Alamatti Dam Thungabhadra
2.
Salal Jhelum
3.
Sardar Sarovar Dam Narmada

Which of the pairs given above is/are correctly matched?


A. 1 only
B. 1 and 3 only
C. 2 and 3 only
D. 3 only

Correct Answer : D

Answer Justification :

The Almatti Dam is a hydroelectric project on the Krishna River in North Karnataka, India
which was completed in July 2005. Hence, pair 1 is incorrectly matched.

Salal Dam, also known as Salal Hydroelectric Power Station, is a run-of-the-river power project
on the Chenab River in the Reasi district of the union territory of Jammu and Kashmir. Hence,
pair 2 is incorrectly matched.

The Sardar Sarovar Dam is a gravity dam on the Narmada river near Navagam, Gujarat in
India. Four Indian states, Gujarat, Madhya Pradesh, Maharastra and Rajasthan, receive water and
electricity supplied from the dam. Hence, pair 3 is correctly matched.

9 Consider the following statements:


1. Lakshadweep group of Islands is divided by ten degree channel.
2. Indian mainland and Sri Lanka are separated by Palk strait.

Which of the statements given above is/are correct?


A. 1 only
B. 2 only

6
Total Marks : 200
Online Prelims TEST - 6 (SUBJECT WISE)
( InsightsIAS Mock Test Series for UPSC Preliminary Exam 2020 )

C. Both 1 and 2
D. Neither 1 nor 2

Correct Answer : B

Answer Justification :

The islands of the Arabian sea include Lakshadweep and Minicoy. These are scattered between
8°N-12°N and 71°E -74°E longitude. These islands are located at a distance of 280 km-480 km off
the Kerala coast. The entire island group is built of coral deposits. There are approximately 36
islands of which 11 are inhabited. Minicoy is the largest island with an area of 453 sq. km. The
entire group
of islands is broadly divided by the Eleventh degree channel, north of which is the Amini
Island and to the south of the Canannore Island. Hence, statement 1 is incorrect.

Sri Lanka, formerly Ceylon, island country lying in theIndian Ocean and separated from
peninsular India by the Palk Strait. Hence, statement 2 is correct.

10 Which of the following rivers is not a tributary of River Krishna?

A. Tungabhadra
B. Koyna
C. Musi
D. Sabari

Correct Answer : D

Answer Justification :

The Tungabhadra River is a river in India that starts and flows through the state of Karnataka
during most of its course, before flowing along the border between Telangana, Andhra Pradesh and
ultimately joining the Krishna River.

The Koyna River is a tributary of the Krishna River which originates in Mahableshwar, Satara
district, western Maharashtra, India.

Musi River is a tributary of the Krishna River in the Deccan Plateau flowing through
Telangana state in India. Hyderabad stands on the banks of Musi river, which divides the historic
old city and the new city.

Sabari River is one of the main tributaries of Godavari. It originates from the western slopes of
Eastern Ghats in Odisha state from Sinkaram hill ranges at 1370 m MSL. It is also known as Kolab
river in Odisha.

Hence, option (d) is correct.

7
Total Marks : 200
Online Prelims TEST - 6 (SUBJECT WISE)
( InsightsIAS Mock Test Series for UPSC Preliminary Exam 2020 )

11 Consider the following statements:


1. The rivers originating from the Amarkantak hills follow the radial pattern.
2. The boundary line separating one drainage basin from the other is known as the watershed.

Which of the statements given above is/are correct?


A. 1 only
B. 2 only
C. Both 1 and 2
D. Neither 1 nor 2

Correct Answer : C

Answer Justification :

When the rivers originate from a hill and flow in all directions, the drainage pattern is known as
‘radial’. The rivers originating from the Amarkantak range present a good example of it.
Hence, statement 1 is correct.

An area drained by a river and its tributaries is called a drainage basin. The boundary line
separating one drainage basin from the other is known as the watershed. Hence,
statement 2 is correct.

12 Consider the following statements:


1. Sentinelese tribes live in the state of Jharkhand.
2. Irulas are a Dravidian ethnic group inhabiting the area of the Nilgiri Mountains.
3. The Gaddis are a tribe living mainly in the Indian state of Himachal Pradesh and the union territory
of Jammu and Kashmir.

Which of the statements given above is/are correct?


A. 1 and 2 only
B. 2 and 3 only
C. 3 only
D. 1, 2 and 3

Correct Answer : B

Answer Justification :

The Sentinelese, also known as the Sentineli and the North Sentinel Islanders, are an
indigenous people who inhabit North Sentinel Island in Andaman and Nicobar Islands froup in
the Bay of Bengal in India. They are considered one of the world's last uncontacted peoples.
Hence, statement 1 is incorrect.

Irula are a Dravidian ethnic group inhabiting the area of the Nilgiri mountains, in the
states of Tamil Nadu and Kerala, India. A scheduled tribe, their population in this region is

8
Total Marks : 200
Online Prelims TEST - 6 (SUBJECT WISE)
( InsightsIAS Mock Test Series for UPSC Preliminary Exam 2020 )

estimated at 25,000 people. People of Irula ethnicity are called Irular, and speak Irula, which
belongs to the Dravidian family. Hence, statement 2 is correct.

The Gaddis are a tribe living mainly in the Indian state of Himachal Pradesh and the union territory
of Jammu and Kashmir. Hence, statement 3 is correct.

13 Which of the following states do not share border with Bangladesh?

A. Mizoram
B. Meghalaya
C. Tripura
D. Manipur

Correct Answer : D

Answer Justification :

Hence, option (d) is correct.

14 Consider the following statements:


1. Satluj river passes through the Kashmir valley below Wular Lake.
2. Dul hasti Hydroelectric plant is built on river Chandra.
3. Godavari river originates in Triambakeshwar, Maharashtra.

Which of the statements given above is/are correct?


A. 1 only
B. 2 and 3 only
C. 3 only

9
Total Marks : 200
Online Prelims TEST - 6 (SUBJECT WISE)
( InsightsIAS Mock Test Series for UPSC Preliminary Exam 2020 )

D. 1, 2 and 3

orrect Answer : B

Answer Justification :

Jhelum River, river of northwestern India and northern and eastern Pakistan. It constitutes the
westernmost of the five rivers of the Punjab region that merge with the Indus River in
eastern Pakistan. The river meanders northwestward from the northern slope of the Pir
Panjal Range through the Valley of Kashmir to Wular Lake at Srinagar, which controls its
flow. Hence, statement 1 is incorrect.

Dul Hasti is a 390 MW hydroelectric power plant in Kishtwar district of Jammu and
Kashmir, India built by NHPC. The power plant is a run-of-the-river type on Chandra River, a
tributary of Chenab River. Hence, statement 2 is correct.

The Godavari is India's second longest river after the Ganga. Its source is in
Triambakeshwar, Maharashtra. It flows east for 1,465 kilometres, draining the states of
Maharashtra, Telangana, Andhra Pradesh, Chhattisgarh, Odisha, ultimately emptying into the Bay
of Bengal . Hence, statement 3 is correct.

15 With reference to the cropping seasons in India, consider the following statements:
1. The Rabi cropping season is from July –October during the south-west monsoon.
2. The rabi crops include wheat, barley, oats (cereals), chickpea/gram (pulses), linseed, mustard
(oilseeds) etc.

Which of the statements given above is/are correct?


A. 1 only
B. 2 only
C. Both 1 and 2
D. Neither 1 nor 2

Correct Answer : B

Answer Justification :

The rabi crops are sown around mid-November, preferably after the monsoon rains are over, and
harvesting begins in April / May. The crops are grown either with rainwater that has percolated into
the ground, or using irrigation. The rabi crops include wheat, barley, oats (cereals), chickpea/gram
(pulses), linseed, mustard (oilseeds) etc. Hence, statement 1 is incorrect and statement 2 is
correct.

16 Rosewood, Mahogony and Ebony are the tree species found in which of the following type of forests?

A. Tropical Evergreen forests

10
Total Marks : 200
Online Prelims TEST - 6 (SUBJECT WISE)
( InsightsIAS Mock Test Series for UPSC Preliminary Exam 2020 )

B. Tropical moist deciduous


C. Tropical thorn forests
D. Montane forests

Correct Answer : A

Answer Justification :

Tropical Evergreen forests are found in the western slope of the Western Ghats, hills of the
northeastern region and the Andaman and Nicobar Islands. They are found in warm and humid
areas with an annual precipitation of over 200 cm and
mean annual temperature above 22oC. Species found in these forests include rosewood,
mahogony, aini, ebony, etc.

Hence, option (a) is correct.

17 Consider the following hills of Peninsular India:


1. Ajanta range
2. Kaimur hills
3. Maikal Hills

What is the correct sequence of the above from the North to South?
A. 1-2-3
B. 2-1-3
C. 1-3-2
D. 2-3-1

Correct Answer : D

Answer Justification :

Self-explanatory . Kindly refer Atlas.

18 Satluj river enters India through which of the following passes?

A. Shipki La
B. Niti Pass
C. Nathu La
D. Mana Pass

Correct Answer : A

Answer Justification :

11
Total Marks : 200
Online Prelims TEST - 6 (SUBJECT WISE)
( InsightsIAS Mock Test Series for UPSC Preliminary Exam 2020 )

Shipki La is a mountain pass and border post on the India-China border. The river Sutlej, which
is called Langqên Zangbo in Tibet, enters India near this pass. The road is an offshoot of the
ancient Silk Road.

Hence, option (a) is correct.

Niti Pass is an international high mountain pass at an elevation of 5.070m (16,633ft) connecting
Uttarakhand, a state in the northern part of India and southern Tibet, an autonomous region
of the People's Republic of China.

Nathu La is a mountain pass in the Himalayas in East Sikkim district. It connects the Indian state
of Sikkim with China's Tibet Autonomous Region. The pass, at 4,310 m above mean sea level,
forms a part of an offshoot of the ancient Silk Road.

Mana Pass is a mountain pass in the Uttarakhand Himalayas on the border between India and
Tibet.

19 Consider the following statements:


1. Tso Moriri is a fresh water lake located Ladakh region.
2. Dal Lake is located in Srinagar.
3. Atapaka Bird Sanctuary is located on Kolleru lake.

Which of the statements given above is/are correct?


A. 2 only
B. 2 and 3 only
C. 1 and 2 only
D. 1 and 3 only

Correct Answer : B

Answer Justification :

Tso Moriri is a lake in the Ladakhi part of the Changthang Plateau in Jammu and Kashmir in
Northern India. The lake and surrounding area are protected as the Tso Moriri Wetland
Conservation Reserve. It is a brackish water lake. Hence, statement 1 is incorrect.

Dal is a lake in Srinagar, the summer capital of Jammu and Kashmir, India. The urban lake, which
is the second largest in the union territory, is integral to tourism and recreation in Kashmir .
Hence, statement 2 is correct.

Atapaka Bird Sanctuary, located in Kolleru Lake on the borders of Krishna and West Godavari
districts, has been identified as the largest spot-billed pelican home in the world. Hence,
statement 3 is correct.

20 Consider the following statements:


1. The ranges of the Eastern Himalayas are more continuous compared to Western Himalayas.

12
Total Marks : 200
Online Prelims TEST - 6 (SUBJECT WISE)
( InsightsIAS Mock Test Series for UPSC Preliminary Exam 2020 )

2. Duns' or 'Doons' formations are characteristic feature of Arunachal Himalayas.


3. The Himachal and Uttarakhand Himalayas lies between the river Jhelum in the west and the Kali in
the east.

Which of the statements given above is/are correct?


A. 1 and 2 only
B. 1 only
C. 1, 2 and 3
D. None

Correct Answer : D

Answer Justification :

The ranges of the Western Himalayas are more continuous compared to Eastern Himalayas. Hence,
statement 1 is incorrect.

The southernmost part of Northwest Himalayan region or Kashmir Himalayan region consists of
longitudinal valleys known as ‘duns’. Jammu dun and Pathankot dun are important examples.
Hence, statement 2 is incorrect.

The Himachal and Uttarakhand Himalayas lies approximately between the Ravi in the west
and the Kali (a tributary of Ghaghara) in the east. It is drained by two major river systems of
India, i.e. the Indus and the Ganga. Tributaries of the Indus include the river Ravi, the Beas and the
Satluj, and the tributaries of Ganga flowing through this region include the Yamuna and the
Ghaghara. Hence, statement 3 is incorrect.

21 Consider the following statements:


1. The Alaknanda has its source in the Satopanth glacier.
2. The Ghaghara originates in the glaciers of Milam.
3. The Sarda or Saryu river rises in the Mapchachungo glacier in the Nepal Himalayas

Which of the statements given above is/are correct?


A. 1 only
B. 2 and 3 only
C. 1 and 3 only
D. 1, 2 and 3

Correct Answer : A

Answer Justification :

At Devprayag, the Bhagirathi meets the Alaknanda; hereafter, it is known as the


Ganga. The Alaknanda has its source in the Satopanth glacier above Badrinath. Hence,
statement 1 is correct.

13
Total Marks : 200
Online Prelims TEST - 6 (SUBJECT WISE)
( InsightsIAS Mock Test Series for UPSC Preliminary Exam 2020 )

The Ghaghara originates in the glaciers of Mapchachungo. After collecting the waters of its
tributaries – Tila, Seti and Beri, it comes out of the mountain, cutting a deep gorge at Shishapani.
The river Sarda (Kali or Kali Ganga) joins it in the plain before it finally meets the Ganga at
Chhapra. Hence, statement 2 is incorrect.

The Sarda or Saryu river rises in the Milam glacier in the Nepal Himalayas where it is known
as the Goriganga. Along the Indo-Nepal border, it is called Kali or Chauk, where it joins the
Ghaghara. Hence, statement 3 is incorrect.

22 Which of the following rivers does/do originate outside the Indian Territory
1. Brahmaputra
2. Sutlej
3. Chenab
4. Ravi

Select the correct answer using the code given below:


A. 1 and 2 only
B. 1, 3 and 4 only
C. 1, 2 and 3 only
D. 3 and 4 only

Correct Answer : A

Answer Justification :

Brahmaputra River, Bengali Jamuna, Tibetan Tsangpo, Chinese (Pinyin) Yarlung Zangbo
Jiang or (Wade-Giles romanization) Ya-lu-tsang-pu Chiang, major river of Central and South Asia.
It flows some 1,800 miles (2,900 km) from its source in the Tibetan Himalayas to
its confluence with the Ganges (Ganga) River, after which the mingled waters of the two rivers
empty into the Bay of Bengal.

The source of the Sutlej is west of Lake Rakshastal in Tibet, as springs in


an ephemeral stream channel descending from this lake.

The river Chenab is formed by the confluence of two rivers, Chandra and Bhaga, at Tandi,
8 km southwest of Kyelang, in the Lahaul and Spiti district in the Indian state of Himachal
Pradesh. The Bhaga river originates from Surya taal lake, which is situated a few kilometers east of
the Bara-lacha la pass in the in Himachal Pradesh. The Chandra river originates from glaciers east
of the same pass.

The river Ravi rises in the Bara Bhangal, District Kangra in Himachal Pradesh, India.

Hence, option (a) is correct.

23 Consider the following hills of Eastern ghats:


1. Palkonda hills

14
Total Marks : 200
Online Prelims TEST - 6 (SUBJECT WISE)
( InsightsIAS Mock Test Series for UPSC Preliminary Exam 2020 )

2. Javadi hills
3. Palani hills

Arrange the above given hills in the direction from South to North:
A. 3-2-1
B. 3-1-2
C. 1-2-3
D. 1-3-2

Correct Answer : A

Answer Justification :

Hence, option (a) is correct.

24 Which of the following regions is/are considered as potential sites for producing tidal energy:
1. The Gulf of Cambay
2. The Gulf of Kutch
3. The Gangetic delta in the Sunderbans region

Select the correct answer using the code given below:


A. 1 only
B. 2 and 3 only
C. 1 and 2 only
D. 1, 2 and 3

Correct Answer : D

Answer Justification :

15
Total Marks : 200
Online Prelims TEST - 6 (SUBJECT WISE)
( InsightsIAS Mock Test Series for UPSC Preliminary Exam 2020 )

As per a recent study, there is an estimated potential of about 8,000 megawatts (MW) of tidal
energy in India.

Out of the 8,000 MW of tidal energy, 7,000 MW is present in the Gulf of Khambhat, 1,200 MW in
the Gulf of Kutch in Gujarat, and about 100 MW in the Gangetic delta in Sunderbans in West
Bengal.

Refer: http://pib.nic.in/PressReleaseIframePage.aspx?PRID=1514971

25 Consider the following pairs:


Nuclear Power Plants Location
1. Kundankulam : Tamil Nadu
2. Jaitapur : Gujarat
3. Narora : Uttar Pradesh

Which of the pairs given above is/are correctly matched?


A. 1, 2 and 3
B. 1 and 3 only
C. 3 only
D. None

Correct Answer : B

Answer Justification :

Kudankulam Nuclear Power Plant is the single largest nuclear power station in India, situated in
Koodankulam in the Tirunelveli district of the state Tamil Nadu. Hence, pair 1 is correctly
matched.

Jaitapur Nuclear Power Project is a proposed nuclear power plant in Maharashtra, India.
Hence, pair 2 is incorrectly matched

Narora Atomic Power Station is a nuclear power plant located in Narora, Bulandshahar District
in Uttar Pradesh, India. Hence, pair 3 is correctly matched

26 The cities of Bina, Numaligarh and Nagapattanam are well-known for which of the following?

A. Thermal Power plants


B. Oil Refineries
C. Bauxite ores
D. Iron and Steel Plants

Correct Answer : B

Answer Justification :

16
Total Marks : 200
Online Prelims TEST - 6 (SUBJECT WISE)
( InsightsIAS Mock Test Series for UPSC Preliminary Exam 2020 )

The cities of Bina, Numaligarh and Nagapattanam are well known for Oil Refineries.

Hence, option (b) is correct.

27 Consider the following statements:


1. Khardung La is a mountain pass in the Ladakh region of the Indian union territory of Ladakh.
2. Terai region is found to the north of the Bhabar region.
3. World’s largest riverine island, Majuli is located in Assam.

Which of the statements given above is/are correct?


A. 1 and 3 only
B. 1 and 2 only
C. 2 and 3 only
D. 1, 2 and 3

Correct Answer : A

Answer Justification :

Khardung La is a mountain pass in the Ladakh region of the Indian union territory of Ladakh. The
pass on the Ladakh Range is north of Leh and is the gateway to the Shyok and Nubra valleys.
Hence, statement 1 is correct.

From the north to the south, Northern Plains can be divided into three major zones: the Bhabar, the
Tarai and the alluvial plains. The alluvial plains can be
further divided into the Khadar and the Bhangar.

South of the Bhabar is the Tarai belt, with an approximate width of 10-20km where most of the
streams and rivers re-emerge without having any properly demarcated channel, thereby, creating
marshy and swampy conditions known as the Tarai. Hence, statement 2 is incorrect.

Majoli is a river island in the Brahmaputra River, Assam and in 2016 it became the first island
to be made a district in India. Hence, statement 3 is correct.

28 What is/are likely reasons for higher frequency of cyclones in Bay of Bengal when compared to
Arabian Sea?
1. Higher surface temperature of Arabian Sea
2. Constant inflow of fresh water from the Ganga and Brahmaputra rivers
3. Higher Rainfall received in Bay of Bengal.

Select the correct answer using the code given below:


A. 1 and 2 only
B. 2 and 3 only
C. 1 and 3 only
D. 1, 2 and 3

17
Total Marks : 200
Online Prelims TEST - 6 (SUBJECT WISE)
( InsightsIAS Mock Test Series for UPSC Preliminary Exam 2020 )

Correct Answer : B

Answer Justification :

Both the Bay of Bengal and Arabian Sea experience cyclonic events. However, when the two are
compared, the Bay of Bengal sees approximately five times as many cyclones as its Western
counterpart. In addition, cyclones in the Bay are stronger and deadlier. And what's more,
nearly 58% of cyclones formed in the Bay of Bengal reach the coast as compared to only 25% of
those formed in the Arabian Sea.

Since sea surface temperatures and humidity both directly correlate with chances of cyclone
formation, the Bay of Bengal is a more likely target because it gets higher rainfall, and
because the sluggish winds around it keep temperatures relatively high: about 28 degrees
around the year. Warm air currents enhance this surface temperature and aid the formation of
cyclones. Hence, statement 1 is incorrect whereas statement 3 is correct.

In addition, the Bay receives higher rainfall and constant inflow of fresh water from the
Ganga and Brahmaputra rivers. This means that the Bay’s surface water keeps getting
refreshed, making it impossible for the warm water to mix with the cooler water below, making it
ideal for a depression. On the other hand, the Arabian Sea receives stronger winds that
help dissipate the heat, and the lack of constant fresh water supply helps the warm water mix with
the cool water, reducing the temperature. Hence, statement 2 is correct.

https://weather.com/en-IN/india/science/news/2018-06-20-bay-of-bengal-cyclone

29 What does the term ‘Bardoli chheerha’ refer to:

A. Local Wind in Assam region


B. Type of Jeera grown in Bardoli region
C. Tribal festival in Manipur
D. Mammal known for its meat.

Correct Answer : A

Answer Justification :

Nor Westers are dreaded evening thunderstorms in Bengal and Assam. Their notorious
nature can be understood from the local nomenclature of ‘Kalbaisakhi’, a calamity of the month of
Baisakh. These showers are useful for tea, jute and rice cultivation. In Assam, these storms are
known as “Bardoli Chheerha”.

Hence, option (a) is correct.

30 Consider the following statements:


1. Bugyals are winter grasslands in the lower reaches of Himalayas.

18
Total Marks : 200
Online Prelims TEST - 6 (SUBJECT WISE)
( InsightsIAS Mock Test Series for UPSC Preliminary Exam 2020 )

2. In the Sunderbans, the mangrove forests are characterized by Heritiera fomes, a species valued for
its timber.

Which of the statements given above is/are correct?


A. 1 only
B. 2 only
C. Both 1 and 2
D. Neither 1 nor 2

Correct Answer : B

Answer Justification :

In the Great Himalayan range, the valleys are mostly inhabited by the Bhotia’s. These are nomadic
groups who migrate to ‘Bugyals’ (the summer glasslands
in the higher reaches) during summer months and return to the valleys during winters. Hence,
statement 1 is incorrect.

In the Sunderbans, the mangrove forests are characterised by Heritiera fomes, a species
valued for its timber. Hence, statement 2 is correct.

31 Sangai Deer is an endangered and endemic species of India found in the Indian State of

A. Manipur
B. Mizoram
C. Meghalaya
D. Tripura

Correct Answer : A

Answer Justification :

The sangai is an endemic and endangered subspecies of brow-antlered deer found only in
Manipur, India. It is also the state animal of Manipur. Its common English name is Manipur
brow-antlered deer or Eld's deer.

Hence, option (a) is correct.

32 Which of the following type of forests covers largest geographical area in India?

A. Tropical Deciduous Forests


B. Montane Forests
C. Tropical thorn forest
D. Tropical wet evergreen forests

19
Total Marks : 200
Online Prelims TEST - 6 (SUBJECT WISE)
( InsightsIAS Mock Test Series for UPSC Preliminary Exam 2020 )

Correct Answer : A

Answer Justification :

Tropical Deciduous Forests are the most widespread forests in India. They are also called the
monsoon forests. They spread over regions which receive rainfall between 70-200 cm. On the basis
of the availability of water, these forests are further divided into moist and dry deciduous.

Hence, option (a) is correct.

33 Consider the following statements:


1. Bangar is the new alluvium and is deposited by floods annually.
2. Khadar represents a system of older alluvium, deposited away from the flood plains.

Which of the statements given above is/are correct?


A. 1 only
B. 2 only
C. Both 1 and 2
D. Neither 1 nor 2

Correct Answer : D

Answer Justification :

The alluvial soils vary in nature from sandy loam to clay. They are generally rich in potash but poor
in phosphorous. In the Upper and Middle Ganga plain, two different types of alluvial soils have
developed, viz. Khadar and Bhangar. Khadar is the new alluvium and is deposited by floods
annually, which enriches the soil by depositing fine silts. Bhangar represents a system of older
alluvium, deposited away from the flood plains. Both the Khadar and Bhangar soils contain
calcareous concretions (Kankars).

Hence, both statement 1 and statement 2 are incorrect.

34 Consider the following statements:


1. The black soil retains the moisture for a very long time.
2. The laterite soils develop in areas with high temperature and high rainfall.
3. Red soil develops on crystalline igneous rocks in areas of low rainfall in the eastern and southern
part of the Deccan Plateau.

Which of the statements given above is/are correct?


A. 1 and 2 only
B. 1 and 3 only
C. 2 and 3 only
D. 1, 2 and 3

20
Total Marks : 200
Online Prelims TEST - 6 (SUBJECT WISE)
( InsightsIAS Mock Test Series for UPSC Preliminary Exam 2020 )

Correct Answer : D

Answer Justification :

The
black soils are generally clayey, deep and impermeable. They swell and become sticky when wet
and shrink when dried. So, during the dry season, these soil develop wide cracks. Thus, there
occurs a kind of ‘self ploughing’. Because of this character of slow absorption and loss of
moisture, the black soil retains the moisture for a very long time, which helps the crops,
especially, the rain fed ones, to sustain even during the dry season. Hence, statement 1 is
correct.

The laterite soils develop in areas with high temperature and high rainfall. These are the
result of intense leaching due to tropical rains. With rain, lime and silica are leached away, and soils
rich in iron oxide and aluminum compound are left behind. Hence, statement 2 is correct.

Red soil develops on crystalline igneous rocks in areas of low rainfall in the eastern and
southern part of the Deccan Plateau. Along the piedmont zone of the Western Ghat, long stretch
of area is occupied by red loamy soil. Yellow and red soils are also found in parts of Odisha and
Chattisgarh and in the southern parts of the middle Ganga plain. The soil develops a reddish colour
due to a wide diffusion of iron in crystalline and metamorphic rocks. It looks yellow when it occurs
in a hydrated form. Hence, statement 3 is correct.

35 Consider the following statements:


1. Jog Falls is created by the Netravati River.
2. Nohkalikai Falls is the tallest plunge waterfall in India located near Cherrapunji.
3. Elephant falls is situated in the state of Maharashtra.

Which of the statements given above is/are correct?


A. 2 only
B. 1, 2 and 3
C. 2 and 3 only
D. 1 and 3 only

Correct Answer : A

Answer Justification :

Jog Falls is created by the Sharavathi River dropping 253 m (830 ft), making it the second-highest
plunge waterfall in India after the Nohkalikai Falls with a drop of 335 m (1100 ft) near
Cherrapunji in Meghalaya. Hence, statement 1 is incorrect whereas statement 2 is correct.

Elephant Falls in the outskirts of Shillong is an important tourist attraction, located in Upper
Shillong area. Elephant Falls is among most popular tourist sites of the city and surrounded with
green vegetation. Hence, statement 3 is incorrect.

21
Total Marks : 200
Online Prelims TEST - 6 (SUBJECT WISE)
( InsightsIAS Mock Test Series for UPSC Preliminary Exam 2020 )

36 With reference to the 2011 census, consider the following statements:


1. Punjab has the lowest sex amongst all Indian states.
2. West Bengal is the most densely populated state amongst all Indian states.
3. Mizoram has the highest female literacy rate amongst all Indian states.

Which of the statements given above is/are correct?


A. 1 and 2 only
B. 2 and 3 only
C. 1, 2 and 3
D. None

Correct Answer : D

Answer Justification :

Haryana has the lowest sex (879) amongst all Indian states. Hence, statement 1 is incorrect.

Bihar is the most densely populated state (1106) amongst all Indian states. Hence, statement 2 is
incorrect.

Kerala has the highest female literacy rate (92.07) amongst all Indian states. Hence, statement 3
is incorrect.

37 Consider the following Indian States:


1. Haryana
2. Punjab
3. Bihar

Which of the states given above has shown decline in the child sex ratio in 2011 when compared to
2001 census?
A. 1 and 2 only
B. 3 only
C. 1, 2 and 3
D. 1 and 3 only

Correct Answer : B

Answer Justification :

Please refer https://www.census2011.co.in/sexratio.php

38 With reference to Indian monsoon, consider the following statements:


1. The breaks in monsoon are related to the movement of the monsoon trough.
2. Withdrawal or the retreat of the monsoon is a more gradual process compared to the onset of
monsoon.

22
Total Marks : 200
Online Prelims TEST - 6 (SUBJECT WISE)
( InsightsIAS Mock Test Series for UPSC Preliminary Exam 2020 )

Which of the statements given above is/are correct?


A. 1 only
B. 2 only
C. Both 1 and 2
D. Neither 1 nor 2

Correct Answer : C

Answer Justification :

The phenomenon associated with the monsoon is its tendency to have ‘breaks’ in
rainfall. Thus, it has wet and dry spells. In other words, the monsoon rains take place only for a few
days at a time. They are interspersed with rainless intervals. These breaks in monsoon are
related to the movement of the monsoon trough. For various reasons, the trough and its
axis keep on moving northward or southward, which determines the spatial distribution of
rainfall.

When the axis of the monsoon trough lies over the plains, rainfall is good in these parts. On the
other hand, whenever the axis shifts closer to the Himalayas, there are longer dry spells in the
plains, and widespread rain occur in the mountainous catchment areas of the Himalayan rivers.

Hence, statement 1 is correct.

During October-November, with the apparent movement of the sun towards the south, the monsoon
trough or the low-pressure trough over the northern plains becomes weaker. This is gradually
replaced by a high-pressure system. The south-west monsoon winds weaken and start
withdrawing gradually. By the beginning of October, the monsoon withdraws from the Northern
Plains.

Hence, statement 2 is correct.

39 Consider the following coastal states:


1. Maharashtra
2. Gujarat
3. Andhra Pradesh
4. Tamil Nadu

Arrange the above given states in the increasing order of the length of their coastline?
A. 1-4-3-2
B. 1-3-4-2
C. 4-1-2-3
D. 4-2-1-3

Correct Answer : A

Answer Justification :

23
Total Marks : 200
Online Prelims TEST - 6 (SUBJECT WISE)
( InsightsIAS Mock Test Series for UPSC Preliminary Exam 2020 )

Hence, option (a) is correct.

40 What is/are the reasons for Tamil Nadu coast to remain dry during the Southwest monsoon?
1. It is situated parallel to the Bay of Bengal branch of southwest monsoon.
2. It lies in the rain shadow area of the Arabian Sea branch of the south-west monsoon.

Select the correct answer using the code given below:


A. 1 only
B. 2 only
C. Both 1 and 2
D. Neither 1 nor 2

Correct Answer : C

Answer Justification :

Tamil Nadu coast remains dry during monsoon season.

There are two factors responsible for it:

(i) The Tamil Nadu coast is situated parallel to the Bay of Bengal branch of
southwest monsoon. Hence, statement 1 is correct.

(ii) It lies in the rain shadow area of the Arabian Sea branch of the south-west
monsoon. Hence, statement 2 is correct.

24
Total Marks : 200
Online Prelims TEST - 6 (SUBJECT WISE)
( InsightsIAS Mock Test Series for UPSC Preliminary Exam 2020 )

41 Consider the following statements:


1. Telangana with the Golconda mines is the only diamond producing State in India.
2. Khetri mines in Rajasthan is known for Copper production.
3. Bailadila Range of Chhattisgarh is known for the production of high grade iron ores.

Which of the statements given above is/are correct?


A. 1 and 2 only
B. 2 and 3 only
C. 1 and 3 only
D. 1, 2 and 3

Correct Answer : B

Answer Justification :

Golconda in Telangana and Panna in Madhya Pradesh are diamond producing regions in India.
Hence, statement 1 is incorrect.

Khetri Nagar, well known for its Copper Project, was built by and is under the control of Hindustan
Copper Limited, a public sector undertaking under the Government of India. Khetri Nagar is also
very well-known with name of 'Copper’. Hence, statement 2 is correct.

Bailadila Range of Chhattisgarh is known for the production of high grade iron ores. Hence,
statement 3 is correct.

42 Consider the following statements:


1. Sikkim, Arunachal Pradesh and West Bengal shares international boundary with three countries.
2. Uttarkhand shares international boundary with China and Nepal.
3. Assam shares international boundary with Bangladesh and Bhutan.

Which of the statements given above is/are correct?


A. 1 and 2 only
B. 2 and 3 only
C. 1 and 3 only
D. 1, 2 and 3

Correct Answer : D

Answer Justification :

Kindly refer Atlas.

25
Total Marks : 200
Online Prelims TEST - 6 (SUBJECT WISE)
( InsightsIAS Mock Test Series for UPSC Preliminary Exam 2020 )

43 Consider the following pairs:


Hill Station State
1. Dalhousie Uttarakhand
2. Wilson Hills Maharashtra
3. Jampui Hills Mizoram

Which of the pairs given above is/are correctly matched?


A. 1, 2 and 3
B. 2 and 3 only
C. 1 only
D. None

Correct Answer : D

Answer Justification :

A beautiful hill station in the state of Himachal Pradesh, Dalhousie is emerged as one of the
highly visited hill stations of India. Hence, pair 1 is incorrectly matched.

Wilson Hills is a hill station in the Indian state of Gujarat. Wilson Hills stands in a densely
forested region close to the Pangarbari Wildlife Sanctuary. Hence, pair 2 is incorrectly matched.

Jampui Hills is a hill range located in the North Tripura district in the north eastern part of the
Indian state of Tripura. Hence, pair 3 is incorrectly matched.

44 Which of the following Tiger reserve is shared between two states?

26
Total Marks : 200
Online Prelims TEST - 6 (SUBJECT WISE)
( InsightsIAS Mock Test Series for UPSC Preliminary Exam 2020 )

A. Palamau
B. Valmiki
C. Kawal
D. Nagarjuna Sagar Srisailam

Correct Answer : D

Answer Justification :

The Palamu Tiger Reserve is one of the nine original tiger reserves in India and the only one in
the state of Jharkhand, India. It forms part of Betla National Park and Palamu Wildlife Sanctuary.

Valmiki National Park, Tiger Reserve and Wildlife Sanctuary is located at the India-Nepal
border in the West Champaran district of Bihar, India on the bank of river Gandak. It is the only
National park in Bihar.

Kawal Tiger Reserve is located at Jannaram mandal of Mancherial District in Telangana state of
India. Govt of India declared Kawal wildlife sanctuary as Tiger Reserve in 2012. The reserve is the
oldest sanctuary in the northern Telangana region of the state.

Nagarjunsagar-Srisailam Tiger Reserve is the largest tiger reserve in India. It is spread over in the
states of Andhra Pradesh and Telangana.

Hence, option (d) is correct.

45 The famous Vembanad Lake is situated in which of the following state?

A. Tamil Nadu
B. Kerala
C. Karnataka
D. Goa

Correct Answer : B

Answer Justification :

Vembanad is the longest lake in India, and the largest lake in the state of Kerala. Hence,
option (b) is correct.

46 Consider the following pairs:


Minerals Mines
1. Bauxite : Lohardaga
2. Mica : Kodarma
3. Coal : Dalli Rajhara

Which of the pairs given above is/are correctly matched?


27
Total Marks : 200
Online Prelims TEST - 6 (SUBJECT WISE)
( InsightsIAS Mock Test Series for UPSC Preliminary Exam 2020 )

A. 1 and 2 only
B. 2 only
C. 1 and 3 only
D. 1, 2 and 3

Correct Answer : A

Answer Justification :

Dalli Rajhara range is known for having high grade iron ore. Hence, pair 3 is incorrectly
matched.

Other two pairs are correctly matched.

47 Consider the following statements:


1. Karewas are useful for the cultivation of Zafran, a local variety of saffron.
2. The Karbi Anglong and the Meghalaya Plateau are extensions of Peninsular block.

Which of the statements given above is/are correct?


A. 1 only
B. 2 only
C. Both 1 and 2
D. Neither 1 nor 2

Correct Answer : C

Answer Justification :

Karewas are the thick deposits of glacial clay and other materials embedded with moraines.
The Kashmir Himalayas are famous for Karewa formations, which are useful for the cultivation of
Zafran, a local variety of saffron. Hence, statement 1 is correct.

Rising from the height of 150 m above the river plains up to an elevation of 600-900 m is the
irregular triangle known as the Peninsular plateau. Delhi ridge in the northwest, (extension of
Aravalis), the Rajmahal hills in the east, Gir range in the west and the Cardamom hills in the south
constitute the outer extent of the Peninsular plateau. However, an extension of this is also seen
in the northeast, in the form of Shillong and Karbi-Anglong plateau. Hence, statement 2 is
correct.

48 Consider the following statements:


1. Approximately 69 per cent of India’s total population lives in villages.
2. Bihar is the least urbanized state amongst all the Indian states.
3. Goa is the most urbanized state amongst all the Indian states.

Which of the statements given above is/are correct?

28
Total Marks : 200
Online Prelims TEST - 6 (SUBJECT WISE)
( InsightsIAS Mock Test Series for UPSC Preliminary Exam 2020 )

A. 1 only
B. 1 and 3 only
C. 1, 2 and 3
D. None

orrect Answer : B

Answer Justification :

Of the 1.21 billion Indians, 833 million (68.84%) live in rural areas while 377 million stay in urban
areas. Hence, statement 1 is correct.

Himachal Pradesh is the least urbanized state amongst all the Indian states. Hence, statement 2 is
incorrect.

Goa is the most urbanized state amongst all the Indian states followed by Mizoram.

Hence, statement 3 is correct.

49 Consider the following statements:


1. In India, highest share of urban population is concentrated in Class II towns.
2. Cities accommodating population size between one to five million are called metropolitan cities.

Which of the statements given above is/are correct?


A. 1 only

29
Total Marks : 200
Online Prelims TEST - 6 (SUBJECT WISE)
( InsightsIAS Mock Test Series for UPSC Preliminary Exam 2020 )

B. 2 only
C. Both 1 and 2
D. Neither 1 nor 2

orrect Answer : B

Answer Justification :

Hence, statement 1 is incorrect.

Urban centre with population of more than one lakh is called a city or class I town. Cities
accommodating population size between one to five million are called metropolitan cities. Hence,
statement 2 is correct.

50 Consider the following statements:


1. About one-fourth of the total cropped area in India is under rice cultivation.
2. In West Bengal farmers grow three crops of rice called ‘aus’, ‘aman’ and ‘boro’.

Which of the statements given above is/are correct?


A. 1 only
B. 2 only
C. Both 1 and 2
D. Neither 1 nor 2

orrect Answer : C

Answer Justification :

Rice is a staple food for the overwhelming majority of population in India. India contributes 21.2 per
cent of rice production in the world and ranked second
after China in 2015. About one-fourth of the total cropped area in the country is under rice

30
Total Marks : 200
Online Prelims TEST - 6 (SUBJECT WISE)
( InsightsIAS Mock Test Series for UPSC Preliminary Exam 2020 )

cultivation. Hence, statement 1 is correct.

In southern states and West Bengal the climatic conditions allow the cultivation of
two or three crops of rice in an agricultural year. In West Bengal farmers grow three crops of
rice called ‘aus’, ‘aman’ and ‘boro’. Hence, statement 2 is correct.

51 Consider the following statements:


1. About 80 per cent of the coal deposits in India is of bituminous type and is of non-coking grade.
2. Jharia coal field is located in Odisha.
3. Lignite type of coal is located in Nyveli, Tamil Nadu.

Which of the statements given above is/are correct?


A. 1 only
B. 1 and 3 only
C. 2 and 3 only
D. 1, 2 and 3

Correct Answer : B

Answer Justification :

Coal is a one of the important minerals which is mainly used in the generation of thermal power and
smelting of iron ore. Coal occurs in rock sequences mainly of two geological ages, namely
Gondwana and tertiary deposits.

About 80 per cent of the coal deposits in India is of bituminous type and is of non-coking grade.
Hence, statement 1 is correct.

Jharia Coalfield is a large coal field located in the east of India in Jharia, Jharkhand. Jharia is the
largest coal field followed by Raniganj. Hence, statement 2 is incorrect.

Besides, the brown coal or lignite occur in the coastal areas of Tamil Nadu (Nyveli),
Puducherry, Gujarat and Jammu and Kashmir. Hence, statement 3 is correct.

52 Consider the following pairs:


Oil field State
1. Lunej Andhra Pradesh
2. Naharkatiya Arunachal Pradesh
3. Digboi Assam

Which of the pairs given above is/are correctly matched?


A. 1 and 2 only
B. 3 only
C. 1 and 3 only
D. 1, 2 and 3

31
Total Marks : 200
Online Prelims TEST - 6 (SUBJECT WISE)
( InsightsIAS Mock Test Series for UPSC Preliminary Exam 2020 )

Correct Answer : B

Answer Justification :

In Assam, Digboi, Naharkatiya and Moran are important oil producing areas. The
major oilfields of Gujarat are Ankaleshwar, Kalol, Mehsana, Nawagam, Kosamba and Lunej

Hence, pair 1 and pair 2 are incorrectly matched whereas pair 3 is correctly matched.

53 Consider the following pairs:


Peaks State
1. Kudremukh Karnataka
2. Kalsubai Maharashtra
3. Doddabetta Tamil Nadu

Which of the pairs given above is/are correctly matched?


A. 1 and 2 only
B. 1 and 3 only
C. 3 only
D. 1, 2 and 3

Correct Answer : D

Answer Justification :

Kuduremukha is a mountain range and name of a peak located in Chikkamagaluru district, in


Karnataka, India. Hence, pair 1 is correctly matched.

Kalsubai is a mountain in the Western Ghats, located in the Indian state of Maharashtra.
Its summit situated at an elevation of 1646 metres is the highest point in Maharashtra The
mountain range lies within the Kalsubai Harishchandragad Wildlife Sanctuary. Hence, pair 2 is
correctly matched.

Doddabetta is the highest mountain in the Nilgiri Mountains at 2,637 metres. It is located in Tamil
Nadu. Hence, pair 3 is correctly matched.

Kindly refer atlas.

54 With reference to the phenomena of Western disturbances in northern plains, consider the following
statements:
1. These are low-pressure systems, originate over the Mediterranean Sea and western Asia and move
into India, along with the westerly flow.
2. They are of immense importance for the cultivation of ‘rabi’ crops. Which of the statements given
above is/are correct?

A. 1 only

32
Total Marks : 200
Online Prelims TEST - 6 (SUBJECT WISE)
( InsightsIAS Mock Test Series for UPSC Preliminary Exam 2020 )

B. 2 only
C. Both 1 and 2
D. Neither 1 nor 2

Correct Answer : C

Answer Justification :

A characteristic feature of the cold weather season over the northern plains is the inflow of cyclonic
disturbances from the west and the northwest. These low-pressure systems, originate over the
Mediterranean Sea and western Asia and move into India, along with the westerly flow.

They cause the much-needed winter rains over the plains and snowfall in the mountains. Although
the total amount of winter rainfall locally known as ‘mahawat’ is small, they are of immense
importance for the cultivation of ‘rabi’ crops.

55 Consider the following pairs:


Indigenous Cattle Breed Originating state
1. Khillari Karnataka
2. Amritmahal Maharashtra
3. Kangayam Tamil Nadu

Which of the pairs given above is/are correctly matched?


A. 1 and 2 only
B. 3 only
C. 1 and 3 only
D. 1, 2 and 3

Correct Answer : B

Answer Justification :

Khillari is drought cattle breed originated from Sholapur and Sitapur districts of Maharashtra.
Hence, pair 1 is incorrectly matched.

Amritmahal is drought cattle breed originated in Hassan, Chikmagalur and Chitradurga district of
Karnataka. Hence, pair 2 is incorrectly matched.

Kangayam is drought cattle breed originated in Kangayam, Dharapuram, Perundurai, Erode,


Bhavani and part of Gobichettipalayam taluk of Erode and Coimbatore district, Tamil Nadu.
Hence, pair 3 is correctly matched.

http://www.agritech.tnau.ac.in/expert_system/cattlebuffalo/Breeds%20of%20cattle%20&%20baffalo
.html

33
Total Marks : 200
Online Prelims TEST - 6 (SUBJECT WISE)
( InsightsIAS Mock Test Series for UPSC Preliminary Exam 2020 )

56 Consider the following pairs:


Wildlife Sanctuary State
1. Eaglenest Wildlife Sanctuary Assam
2. Sonai Rupai Arunachal Pradesh
3. Lengteng Manipur

Which of the pairs given above is/are NOT correctly matched?


A. 1 and 2 only
B. 2 and 3 only
C. 1 only
D. 1, 2 and 3

Correct Answer : D

Answer Justification :

Eaglenest or Eagle's Nest Wildlife Sanctuary is a protected area of India in the Himalayan
foothills of West Kameng District, Arunachal Pradesh. Hence, pair 1 is incorrectly matched.

Sonai Rupai Wildlife Sanctuary is a protected area located in the state of Assam. Hence, pair 2
is incorrectly matched.

Lengteng Wildlife Sanctuary is a protected area in Champhai district in eastern Mizoram.


Hence, pair 3 is incorrectly matched.

57 Which of the following are tributaries of Brahmaputra river?


1. Dibang
2. Subansiri
3. Raidak

Select the correct answer using the code given below:


A. 1 and 2 only
B. 2 and 3 only
C. 1 and 3 only
D. 1, 2 and 3

Correct Answer : D

Answer Justification :

Dibang River, also known as Sikang in Adi and Talon in Idu, is a tributary river of the
Brahmaputra that originates and flows through the Mishmi Hills and northeast India from the
state of Arunachal Pradesh.

The Subansiri River is a tributary of the Brahmaputra River in the Indian states of Assam and
Arunachal Pradesh, and the Tibet Autonomous Region of China. The Subansiri is the largest

34
Total Marks : 200
Online Prelims TEST - 6 (SUBJECT WISE)
( InsightsIAS Mock Test Series for UPSC Preliminary Exam 2020 )

tributary of the Brahmaputra.

The Raidāk River, also called Wong Chhu in Bhutan, is a tributary of the River Brahmaputra,
and a trans-boundary river. It flows through Bhutan, India and Bangladesh.

Hence, option (d) is correct.

58 Which of the following rivers in India is/are west flowing rivers?


1. Narmada
2. Brahmani
3. Mahi
4. Sabarmati

Select the correct answer using the code given below:


A. 1, 2 and 4 only
B. 1, 3 and 4 only
C. 2, 3 and 4 only
D. 1, 2, 3 and 4

Correct Answer : B

Answer Justification :

Narmada river is the fifth longest river in the Indian subcontinent and one of the major river that
draining through the Gulf of Cambay into the Arabian Sea.

The Brahmani is a major seasonal river in the Odisha state of Eastern India. The Brahmani is
formed by the confluence of the Sankh and South Koel rivers. It discharges into Bay of Bengal.

Mahi River originates in Madhya Pradesh and discharge into Arabian Sea by creating an
wide estuary past Bay of Khambhat.

Sabarmati River originates in Dhebar lake near Udaipur in Aravalli hills of Rajasthan and
meets the Arabian Sea at Gulf of Cambay. Ahmedabad and Gandhinagar city of Gujarat are
situated on the banks of Sabarmati river.

Hence, option (b) is correct.

59 Which of the following National Waterways are NOT correctly matched?


National waterway Stretch
1. NW-2 : Kottapuram-Kollam
2. NW-3 : Sadiya-Dhubri
3. NW-1 : Allahabad- Haldia

Select the correct answer using the code given below:


A. 1 and 2 only
B. 3 only

35
Total Marks : 200
Online Prelims TEST - 6 (SUBJECT WISE)
( InsightsIAS Mock Test Series for UPSC Preliminary Exam 2020 )

C. 1, 2 and 3
D. None

orrect Answer : B

Answer Justification :

60 Culturable waste land is a category of land use in India. It means the


land which

A. is used only once in an agricultural season and rest of the time as pasture lands
B. Has not been sown more than once during periods of drought
C. Has the potential to be used for agriculture but is not being used for cultural reasons
D. is left uncultivated for more than five agricultural years

Correct Answer : D

Answer Justification :

Culturable waste land is left uncultivated for more than 5 agricultural years.

36
Total Marks : 200
Online Prelims TEST - 6 (SUBJECT WISE)
( InsightsIAS Mock Test Series for UPSC Preliminary Exam 2020 )

It is a category of uncultivable land. Hence, option (d) is correct.

61 With reference to Small Farmers’ Agri-Business Consortium (SFAC) in India, consider the following
statements:
1. SFAC is constituted as a society to facilitate agri-business ventures by catalyzing private
investment.
2. SFAC implements National Agriculture Market (e-NAM) Electronic Trading platform.
3. SFAC is one of the procurement agency similar to Food Corporation of India.

Which of the statements given above is/are correct?


A. 1 and 2 only
B. 2 and 3 only
C. 1 only
D. 1, 2 and 3

Correct Answer : D

Answer Justification :

Small Farmers’ Agri-Business Consortium (SFAC) as a Society in 1994 to facilitate agri-


business ventures by catalyzing private investment through Venture Capital Assistance (VCA)
Scheme in close association with financial institutions. Hence, statement 1 is correct.

The setting up of State level SFAC as counterpart agency of Central SFAC for agribusiness projects
was part of the Scheme.

The main functions of SFAC are:

1. Promotion of development of small agribusiness through VCA scheme;

2. Helping formation and growth of Farmer Producer Organizations (FPOs) / Farmer Producer
Companies (FPCs);

3. (Improving availability of working capital and development of business activities of


FPOs/FPCs through Equity Grant and Credit Guarantee Fund Scheme;

4. Implementation of National Agriculture Market (e-NAM) Electronic Trading platform.


Hence, statement 2 is correct.

Along with that, SFAC is one of the Central Procurement Agencies for pulses and oilseeds
under Price Stabilisation Fund of Department of Consumer Affairs. Hence, statement 3 is
correct.

37
Total Marks : 200
Online Prelims TEST - 6 (SUBJECT WISE)
( InsightsIAS Mock Test Series for UPSC Preliminary Exam 2020 )

http://pib.nic.in/newsite/PrintRelease.aspx?relid=169745

62 Consider the following statements:


1. Neem coated urea releases neem oil in the soil which increases nitrogen fixation by the soil
microorganisms.
2. Certification of seeds is mandatory in India.
3. Zero budget farming practice believes in no addition of synthetic fertilizers and pesticides into the
soil.

Which of the statements given above is/are correct?


A. 3 only
B. 1 and 3 only
C. 2 and 3 only
D. 1 and 2 only

Correct Answer : A

Answer Justification :

Neem coated Urea slows down the rate of dissolution of urea in the soil. However neem oil
released by neem coated doesn’t help in increasing nitrogen fixation by the soil microorganisms.
Hence, statement 1 is incorrect.

Seed certification is a process designed to maintain and make available to the general public
continuous supply of high quality seeds and propagating materials of notified kinds and varieties of
crops, so grown and distributed to ensure the physical identity and genetic purity.

Seed certification is a legally sanctioned system for quality control of seed multiplication and
production.

The main objective of the Seed Certification is to ensure the acceptable standards of seed viability,
vigour, purity and seed health.

In India, Seeds Certification shall be conducted by the Certification Agency notified under
Section 8 of the Seeds Act, 1966 and it is not mandatory.

Hence, statement 2 is incorrect.

Zero Budget Natural Farming (ZBNF) is a farming practice that believes in natural growth of crops
without adding any fertilizers and pesticides or any other foriegn elements. The word Zero
Budget refers to the zero net cost of production of all crops (inter crops, border crops,
multi crops). The inputs used for seed treatments and other inocluations are locally available in
the form of cowdung and cow urine.

A ZBNF practicisng farmer has lower cost of inputs and thus has better capacity to increase the
incomes. At the same time, ZBNF crops helps in retaining soil. Hence, statement 3 is correct.

38
Total Marks : 200
Online Prelims TEST - 6 (SUBJECT WISE)
( InsightsIAS Mock Test Series for UPSC Preliminary Exam 2020 )

63 Consider the following pairs


Food Grain Largest Producer
1. Rice : Uttar Pradesh
2. Wheat : Punjab
3. Maize : Karnataka

Which of the statements given above is/are correct?


A. 1 and 2 only
B. 2 and 3 only
C. 3 only
D. None

Correct Answer : C

Answer Justification :

IMAGE

Hence, option (c) is correct.

64 Consider the following statements :


1. West Bengal is the leading producer of fisheries followed by Andhra Pradesh
2. About 25% of the marine fish landings are in the west coast and around 75% is contributed by east
coast.
3. Tunas are highly migratory and commercially important fresh water fish species.

Which of the statements given above is/are correct?


A. 1 and 2 only
B. 2 and 3 only
C. 1 and 3 only
D. None

Correct Answer : D

Answer Justification :

Andhra Pradesh is the leading producer of fisheries.

Distribution of Fish Production in India

1. AP

2. WB

39
Total Marks : 200
Online Prelims TEST - 6 (SUBJECT WISE)
( InsightsIAS Mock Test Series for UPSC Preliminary Exam 2020 )

3. Gujarat

4. Kerala

5. Tamil Nadu

6. Maharahtra

Hence, statement 1 is incorrect.

It is estimated that about 75% of the marine fish landings are in the west coast and only 25% is
contributed by east coast. Hence, statement 2 is incorrect.

Important fishes caught are: shark, sardine, herring, anchovies, mackerel and Indian Salmon.

India’s offshore and deep sea fish catch is very poor considering only 10% of the marine potential is
caught at present

Note: Inland Fisheries Production > Marine Fisheries Production

Tuna is an important commercial saltwater fish. Tuna fish is low in calories but high in protein,
niacin, selenium and vitamin B12. Eating tuna can improve weight loss, reduce inflammation.
Hence, statement 3 is incorrect.

65 Consider the following pairs:


Region Crop
1. Shivamoga Arecanut
2. Pampore Saffron
3. Coromondal Soyabean

Which of the pairs given above is/are correctly matched?


A. 1 and 2 only
B. 1 only
C. 1, 2 and 3
D. None

Correct Answer : A

Answer Justification :

Shivamoga district in Karnataka is famous for Arecanut production. Hence, pair 1 is correctly
matched.

Pampore or Pampur is a historic town situated on the eastern side of river Jehlum on Srinagar-
Jammu National Highway in Jammu and Kashmir. It is worldwide famous for its Saffron, so known

40
Total Marks : 200
Online Prelims TEST - 6 (SUBJECT WISE)
( InsightsIAS Mock Test Series for UPSC Preliminary Exam 2020 )

as " Saffron Town of Kashmir". Hence, pair 2 is correctly matched.

Soybean is not widely cultivated in the Coromondal region. Hence, pair 3 is incorrectly
matched.

66 The Visakhapatnam-Chennai Industrial Corridor is being funded by which of the following external
agency?

A. Asian Development Bank (ADB)


B. World Bank(WB)
C. Asian Infrastructure investment Bank(AIIB)
D. New Development Bank(NDB)

Correct Answer : A

Answer Justification :

Asian Development Bank (ADB) has approved US $631 million, in loans and grants, for VCIC,
comprising a Multi-tranche Financing Facility.

Four nodes namely: Visakhapatnam, Kakinada, Kankipadu-Gannavaram and Yerpedu-


Srikalahasti of Andhra Pradesh, have been identified for development in Vishakhapatnam-
Chennai Industrial Corridor (VCIC) on the basis of Conceptual Development Plan (CDP) prepared by
ADB.

http://pib.nic.in/newsite/PrintRelease.aspx?relid=180840

67 With reference to ‘National Policy on Biofuels-2018’ which was recently


approved by the cabinet, consider the following statements:
1. With a thrust on Advanced Biofuels, the Policy indicates a viability gap funding scheme for 2G
ethanol Bio refineries in addition to additional tax incentives, higher purchase price as compared to
1G biofuels.
2. The Policy encourages setting up of supply chain mechanisms for biodiesel production from non-
edible oilseeds, Used Cooking Oil, short gestation crops.

Which of the statement(s) given above is/are correct?


A. 1 only
B. 2 only
C. Both 1 and 2
D. Neither 1 nor 2

Correct Answer : C

Answer Justification :

41
Total Marks : 200
Online Prelims TEST - 6 (SUBJECT WISE)
( InsightsIAS Mock Test Series for UPSC Preliminary Exam 2020 )

The Union Cabinet, chaired by the Prime Minister Shri Narendra Modi has approved National Policy
on Biofuels – 2018.

Salient Features:

1. The Policy categorizes biofuels as "Basic Biofuels" viz. First Generation (1G) bioethanol &
biodiesel and "Advanced Biofuels" - Second Generation (2G) ethanol, Municipal Solid
Waste (MSW) to drop-in fuels, Third Generation (3G) biofuels, bio-CNG etc. to enable
extension of appropriate financial and fiscal incentives under each category.

2. The Policy expands the scope of raw material for ethanol production by allowing use of
Sugarcane Juice, Sugar containing materials like Sugar Beet, Sweet Sorghum, Starch
containing materials like Corn, Cassava, Damaged food grains like wheat, broken rice, Rotten
Potatoes, unfit for human consumption for ethanol production.

3. Farmers are at a risk of not getting appropriate price for their produce during the surplus
production phase. Taking this into account, the Policy allows use of surplus food grains for
production of ethanol for blending with petrol with the approval of National Biofuel
Coordination Committee.

4. With a thrust on Advanced Biofuels, the Policy indicates a viability gap funding
scheme for 2G ethanol Bio refineries of Rs.5000 crore in 6 years in addition to
additional tax incentives, higher purchase price as compared to 1G biofuels. Hence,
statement 1 is correct.

5. The Policy encourages setting up of supply chain mechanisms for biodiesel


production from non-edible oilseeds, Used Cooking Oil, short gestation crops. Hence,
statement 2 is correct.

6. Roles and responsibilities of all the concerned Ministries/Departments with respect to biofuels
has been captured in the Policy document to synergize efforts.

68 Consider the following agriculture practices


1. Zero-Tillage helps in carbon sequestration and requires minimum labour compare to traditional
farming method.
2. Mixed Farming refers to growing two or more crops in the same farmland
3. Vertical farming is the practice of producing food and medicine in vertically stacked layers in
structures like skyscrapers and warehouse.

Which of the statements given above is/are correct?


A. 1, 2 and 3
B. 2 and 3 only
C. 2 only

42
Total Marks : 200
Online Prelims TEST - 6 (SUBJECT WISE)
( InsightsIAS Mock Test Series for UPSC Preliminary Exam 2020 )

D. 1 and 3 only

Correct Answer : D

Answer Justification :

Zero tillage is the process where the crop seed will be sown through drillers without prior land
preparation and disturbing the soil where previous crop stubbles are present.

Zero tillage not only reduce the cost of cultivation it also reduces the soil erosion, crop
duration and irrigation requirement and weed effect which is better than tillage. Zero Tillage (ZT)
also called No Tillage or Nil Tillage. Hence, statement 1 is correct.

Mixed farming is a type of farming which involves both the growing of crops as well as the
raising of livestock. Hence, statement 2 is incorrect.

Intercropping is a multiple cropping practice involving growing two or more crops in


proximity.

Vertical farming is the practice of growing produce in vertically stacked layers. The practice can
use soil, hydroponic or aeroponic growing methods. Vertical farms attempt to produce food in
challenging environments, like where arable land is rare or unavailable. Hence, statement 3 is
correct.

69 Which of the following state launched Rythu Bandhu Scheme to support farmer’s investment for two
crops a year?

A. Karnataka
B. Tamil Nadu
C. Andhra Pradesh
D. Telangana

Correct Answer : D

Answer Justification :

Rythu Bandhu Scheme is proposed by Government of Telangana for providing Investment Support
Agriculture and Horticulture crops by way of grant of Rs. 4,000/- per acre per farmer each season
for purchase of inputs like Seeds, Fertilizers, Pesticides, Labour and other investments in the field
operations of Farmer’s choice for the crop season.

Hence, option (d) is correct.

70 Consider the following statements regarding the cultivation


1. Cotton is a Kharif crop which requires around yearlong frost free days for good harvest.

43
Total Marks : 200
Online Prelims TEST - 6 (SUBJECT WISE)
( InsightsIAS Mock Test Series for UPSC Preliminary Exam 2020 )

2. Tea cultivation requires loamy soil which is slightly acidic in nature and rich in organic matter.
3. International crop research institute for semi-arid tropics is located at Hyderabad.

Which of the statements given above is/are correct?


A. 2 only
B. 1 and 3 only
C. 2 and 3 only
D. 1, 2 and 3

Correct Answer : C

Answer Justification :

Cotton is a Kharif crop which requires around 200 frost free days and for good harvest over
300 days of frost free days. Hence, statement 1 is incorrect.

Tea is a tropical and sub-tropical plant and thrives well in hot and humid climate.

There is a very close relation between climate, the yield and the quality of tea. The ideal
temperature for its growth is 20°-30°C and temperatures above 35°C and below 10°C are harmful
for the Tea. It requires 150-300 cm annual rainfall which should be well distributed throughout
the year.

It also requires loamy soil which is slightly acidic in nature and rich in organic matter.
Hence, statement 2 is correct.

The International Crops Research Institute for the Semi-Arid Tropics (ICRISAT) is a non-
profit, non-political organization that conducts agricultural research for development in the dry
lands of Asia and sub-Saharan Africa. Its headquarter is located at Hyderabad.

Hence, statement 3 is correct.

71 Which of the following fibres is/are man-made fibres?


1. Linen
2. Nylon
3. Polyster

Select the correct answer using the codes below:


A. 1 and 2 only
B. 2 and 3 only
C. 1 and 3 only
D. 1, 2 and 3

Correct Answer : B

Answer Justification :

44
Total Marks : 200
Online Prelims TEST - 6 (SUBJECT WISE)
( InsightsIAS Mock Test Series for UPSC Preliminary Exam 2020 )

Fibres are the raw material of textile industry. Natural fibres are obtained from wool, silk, cotton,
linen and jute and Man-made fibres include nylon, polyester, acrylic and rayon

Hence, option (b) is correct.

72 The expression “Green Gold Revolution ” refers to which of the following?

A. Afforestation program
B. Wind Turbines
C. Bamboo
D. Solar Panels

Correct Answer : C

Answer Justification :

The expression “green gold” has been coined to refer to bamboo because it is a miracle
plant which is often placed in the category of grass product rather than forest product and
matures faster than its other peers.

https://timesofindia.indiatimes.com/good-governance/assam/DrJitendraSingh-called-for-a-Green-Gol
d-revolution/articleshow/54561051.cms

73 Naphthalene balls are obtained from


#00000

A. Crude Petroleum
B. Coal tar
C. Iron ores
D. Plant barks

Correct Answer : B

Answer Justification :

Naphthalene balls used to repel moths and other insects are also obtained from coal tar. Hence,
option (b) is correct.

74 While extracting petroleum from reservoir rocks, what would be the correct sequence in which the
following are encountered in the process?
1. Natural Gas
2. Water
3. Oil

45
Total Marks : 200
Online Prelims TEST - 6 (SUBJECT WISE)
( InsightsIAS Mock Test Series for UPSC Preliminary Exam 2020 )

Select the correct answer using the codes below:


A. 1-2-3
B. 2-1-3
C. 1-3-2
D. 3-1-2

orrect Answer : C

Answer Justification :

Hence, option (c) is correct.

75 Tidal energy is based on which of the following?

A. Difference in temperature of tides


B. Difference in height of tides
C. Marine upwelling
D. Difference in the height of waves

Correct Answer : B

Answer Justification :

Energy generated from tides is called tidal energy. Tidal energy can be harnessed by building
dams at narrow openings of the sea. During high tide the energy of the tides is used to turn the

46
Total Marks : 200
Online Prelims TEST - 6 (SUBJECT WISE)
( InsightsIAS Mock Test Series for UPSC Preliminary Exam 2020 )

turbine installed in the dam to produce electricity. Hence, option (b) is correct.

76 Why are the western slopes of the Western Ghats covered with thick forests and not the eastern
slopes?
1. Soil in eastern slope cannot support vegetation due to absence of nutrients.
2. Eastern side falls in rain shadow zone
3. No protected forests exist on the eastern slopes.

Select the correct answer using the codes below:


A. 1 and 2 only
B. 2 only
C. 1 and 3 only
D. 1, 2 and 3

Correct Answer : B

Answer Justification :

When moisture laden winds cause rainfall in western slopes, upon moving to eastern sides they lose
much of their moisture. Rainfall is less, and thus vegetation is less thick at eastern slopes. Hence,
statement 1 is incorrect whereas statement 2 is correct.

Nilgiri Biosphere Reserve extends to Eastern Slopes. Hence, statement 3 is incorrect.

77 Which of the following rivers do not form ‘deltas’?


1. Tapi
2. Kaveri
3. Narmada
4. Krishna

Select the correct answer using the codes below:


A. 1 and 2 only
B. 2 and 3 only
C. 3 only
D. 1 and 3 only

Correct Answer : D

Answer Justification :

The Narmada and the Tapi are the only long rivers, which flow west and make estuaries. They do
not form deltas.

Most of the major rivers of the Peninsula such as the Mahanadi, the Godavari, the Krishna and
the Kaveri flow eastwards and drain into the Bay of Bengal. These rivers make deltas at

47
Total Marks : 200
Online Prelims TEST - 6 (SUBJECT WISE)
( InsightsIAS Mock Test Series for UPSC Preliminary Exam 2020 )

their mouths.

Hence, option (d) is correct.

Please refer Atlas

78 With reference to drainage pattern, consider the following statement:


1. Dendritic pattern develops where the river channel follows the slope of the terrain.
2. Trellis pattern develops on a strong jointed rocky terrain.
3. A rectangular pattern develops where hard and soft rocks exits parallel to each other.
4. Radial pattern develops when stream flow in different direction from a central peak

Which of the statements given above is/are correct?


A. 1 and 4 only
B. 2 and 3 only
C. 1 and 3 only
D. 1, 2, 3 and 4

orrect Answer : A

Answer Justification :

79 Which of the following rivers flow in rift valley?


1. Luni
2. Damodar
3. Tapi
4. Betwa

Select the correct answer using the codes given below:


A. 1, 2 and 3 only
B. 1 , 3 and 4 only
C. 2 and 3 only
D. 1 , 2, and 4 only

orrect Answer : C

48
Total Marks : 200
Online Prelims TEST - 6 (SUBJECT WISE)
( InsightsIAS Mock Test Series for UPSC Preliminary Exam 2020 )

Answer Justification :

Damodar River, Tapti and Naramada are major rivers that flow in the rift valley.

80 As per the recent census report, which of the following states has /have no Scheduled Tribal
population?
1. Daman and Diu
2. Haryana
3. Punjab
4. Dadra Nagar Haveli

Select the correct answer using the codes given below


A. 1 and 2 only
B. 2 and 3 only
C. 1, 2 and 3 only
D. 1, 2, 3 and 4

Correct Answer : B

Answer Justification :

https://www.census2011.co.in/scheduled-tribes.php

81 Consider the following statements


1. The black rhinoceros is a species of rhinoceros native to eastern and southern Africa.
2. White rhinoceros are naturally found in India
3. Palamau hosts highest number of Indian rhinoceros in India.

Which of the statements given above is/are correct?


A. 1 and 2 only
B. 2 and 3 only
C. 1 only

49
Total Marks : 200
Online Prelims TEST - 6 (SUBJECT WISE)
( InsightsIAS Mock Test Series for UPSC Preliminary Exam 2020 )

D. 1 and 3 only

Correct Answer : C

Answer Justification :

The black rhinoceros or hook-lipped rhinoceros is a species of rhinoceros, native to eastern and
southern Africa including Botswana, Kenya, Malawi, Mozambique, Namibia, South Africa,
Swaziland, Tanzania, Zambia, and Zimbabwe. Although the rhinoceros is referred to as black,
its colors vary from brown to grey. Hence, statement 1 is correct.

The northern white rhinoceros, or northern square-lipped rhinoceros is one of two subspecies of
the white rhinoceros (the other being the southern white rhinoceros).

Formerly found in several countries in East and Central Africa south of the Sahara, this
subspecies is a grazer in grasslands and savanna woodlands. As of March 19, 2018, there
were only two known rhinos of this subspecies left, both of which are female; barring the existence
of unknown or misclassified male northern white rhinos elsewhere in Africa, this makes the
subspecies functionally extinct. They are not naturally found in India. Hence, statement 2 is
incorrect

Kaziranga National Park is a national park in the Golaghat and Nagaon districts of the state of
Assam, India. The sanctuary, which hosts two-thirds of the world's great one-horned
rhinoceroses, is a World Heritage Site.

According to the census held in March 2018 which was jointly conducted by the Forest Department
of the Government of Assam and some recognized wildlife NGOs, the rhino population in Kaziranga
National Park is 2,413.

Hence, statement 3 is incorrect.

https://www.indiatoday.in/trending-news/story/assam-floods-rescue-team-saves-rhino-calf-from-drow
ning-in-kaziranga-hats-off-says-internet-1570401-2019-07-17

82 With reference to the Press Council of India, consider the following statements
1. It is a statutory body in India that governs the conduct of the print media.
2. It encourages the growth of a sense of responsibility and public service among all those engaged in
the profession of journalism.
3. The Press Council of India accepts complaints against and by the press in matters relating to a
journalist's or media organization's ethical failures.

Which of the statements given above is/are correct?


A. 1 and 2 only
B. 2 and 3 only
C. 3 only
D. 1, 2 and 3

50
Total Marks : 200
Online Prelims TEST - 6 (SUBJECT WISE)
( InsightsIAS Mock Test Series for UPSC Preliminary Exam 2020 )

Correct Answer : D

Answer Justification :

The Press Council of India is a statutory body in India that governs the conduct of the print
media. It is  headed by a Chairman, usually a retired judge of the Supreme Court of India. It
consists of 28 other members of whom 20 represent the press and are nominated by the press
organizations/news agencies. Hence, statement 1 is correct.

Some of the functions of Council:

• to help newspapers to maintain their independence

• to build up a code of conduct for newspapers and journalists in accordance with high professional
standards

• to ensure on the part of newspapers and journalists the maintenance of high standards of public
taste and foster a due sense of both the rights and responsibilities of citizenship

• to encourage the growth of a sense of responsibility and public service among all those
engaged in the profession of journalism. Hence, statement 2 is correct.

• to keep under review any development likely to restrict the supply and dissemination of news of
public interest and importance

• to keep under review such cases of assistance received by any newspaper or news agency in India
from foreign sources, as are referred to it by the Central Government.

The Press Council of India accepts complaints against and by the press in matters relating to
a journalist's or media organization's ethical failures. Hence, statement 3 is correct.

http://presscouncil.nic.in/Content/29_3_History.aspx

83 With reference to the Alternative Investment Fund, consider the following statements:
1. There is no official definition for Alternative Investment Fund in India
2. AIF includes Venture Capital Fund, Hedge funds and Private equity funds
3. AIF cannot make invitation to the public at large to subscribe its units and can raise funds from the
sophisticated investors only through private placement

Which of the statements given above is/are correct?


A. 1 and 2 only
B. 2 only
C. 2 and 3 only
D. None

Correct Answer : C

51
Total Marks : 200
Online Prelims TEST - 6 (SUBJECT WISE)
( InsightsIAS Mock Test Series for UPSC Preliminary Exam 2020 )

Answer Justification :

In India, alternative investment funds (AIFs) are defined in Regulation 2(1)(b) of


Securities and Exchange Board of India (Alternative Investment Funds) Regulations, 2012.
Hence, statement 1 is incorrect.

It refers to any privately pooled investment fund, (whether from Indian or foreign sources), in the
form of a trust or a company or a body corporate or a Limited Liability Partnership (LLP) which are
not presently covered by any Regulation of SEBI governing fund management (like, Regulations
governing Mutual Fund or Collective Investment Scheme) nor coming under the direct regulation of
any other sectoral regulators in India-IRDA, PFRDA, RBI.

AIFs includes venture Capital Fund, hedge funds, private equity funds, commodity funds,
Debt Funds, infrastructure funds, etc., while, it excludes Mutual funds or collective investment
Schemes, family trusts, Employee Stock Option / purchase Schemes, employee welfare trusts or
gratuity trusts, ‘holding companies’ within the meaning of Section 4 of the Companies Act, 1956,
securitization trusts regulated under a specific regulatory framework and funds managed by
securitization company or reconstruction company which is registered with the RBI under Section 3
of the Securitization and Reconstruction of Financial Assets and Enforcement of Security Interest
Act, 2002. Hence, statement 2 is correct.

No scheme of an AIF (other than angel fund) shall have more than 1000 investors. (Please note that
the provisions of the Companies Act, 1956 shall apply to the AIF if it is formed as a company). In
case of an angel fund, no scheme shall have more than forty-nine angel investors. However, an AIF
cannot make invitation to the public at large to subscribe its units and can raise funds
from the sophisticated investors only through private placement. Hence, statement 3 is
correct.

https://www.sebi.gov.in/sebi_data/attachdocs/1471519155273.pdf

http://www.arthapedia.in/index.php?title=Alternative_Investment_Funds_(AIFs)

84 Bomkai, Kantha and Ikat saris, sometime seen in the news, is largely found in

A. Maharashtra
B. Odisha
C. Andhra Pradesh
D. Gujarat

Correct Answer : B

Answer Justification :

Bomkai Sari is a handloom saree from Odisha, India. It is an origin of Bomkai, Ganjam district
in the state and is primarily produced by the “Bhulia” community of Subarnapur district. Bomkai is
one of the identified Geographical Indications of India.

52
Total Marks : 200
Online Prelims TEST - 6 (SUBJECT WISE)
( InsightsIAS Mock Test Series for UPSC Preliminary Exam 2020 )

Hence, option (b) is correct.

https://www.thehindu.com/news/national/cooperative-helps-take-tribal-crafts-to-amazons-global-mar
ketplace/article28235876.ece

https://www.thehindu.com/news/national/cooperative-helps-take-tribal-crafts-to-amazons-global-mar
ketplace/article28235876.ece

85 Consider the following statements


1. INS Viraat was the flagship of the Indian Navy before INS Vikramaditya was commissioned in 2013.
2. INS Vikramaditya is the first nuclear powered Indian aircraft carrier.

Which of the statements given above is/are correct?


A. 1 only
B. 2 only
C. Both 1 and 2
D. Neither 1 nor 2

Correct Answer : A

Answer Justification :

INS Viraat was a Centaur-class aircraft carrier of the Indian Navy. INS Viraat was the
flagship of the Indian Navy before INS Vikramaditya was commissioned in 2013.The ship was
completed and commissioned in 1959 as the Royal Navy's HMS Hermes, and decommissioned in
1984. Hence, statement 1 is correct.

INS Vikramaditya is a modified Kiev-class aircraft carrier and the flagship of the Indian Navy, which
entered into service in 2013. She has been renamed in honour of Vikramaditya, a legendary
emperor of India.

It is powered by diesel Engine driven pressurized boilers. On 14 June 2014, the Prime Minister
of India formally inducted INS Vikramaditya into the Indian Navy and dedicated her to the nation.
Hence, statement 2 is incorrect.

https://www.thehindu.com/news/national/finally-aircraft-carrier-viraat-to-be-scraped/article2825204
8.ece

86 Consider the following statements


1. Incremental capital output ratio explains the relationship between the level of investment made in
the economy and the consequent increase in GDP.
2. Lower capital output ratio shows productivity of capital and technological progress

Which of the statements given above is/are correct?


A. 1 only
B. 2 only

53
Total Marks : 200
Online Prelims TEST - 6 (SUBJECT WISE)
( InsightsIAS Mock Test Series for UPSC Preliminary Exam 2020 )

C. Both 1 and 2
D. Neither 1 nor 2

Correct Answer : C

Answer Justification :

The incremental capital output ratio (ICOR) is a frequently used tool that explains the
relationship between the level of investment made in the economy and the consequent
increase in GDP. ICOR indicates the additional unit of capital or investment needed to produce an
additional unit of output.

Hence, statement 1 is correct.

A lower capital output ratio shows that only low level of investment is needed to produce a
given growth rate in the economy. This is considered as a desirable situation. Lower capital
output ratio shows that capital is very productive or efficient. Hence, lower capital output ratio
shows productivity of capital and technological progress. Hence, statement 2 is correct.

https://www.investopedia.com/terms/i/icor.asp

87 Consider the following statements


1. North Korea shares border with China, South Korea and Russia only
2. 49th parallel, popular name given to latitude 49° N that in East Asia roughly
demarcates North Korea and South Korea.

Which of the statements given above is/are correct?


A. 1 only
B. 2 only
C. Both 1 and 2
D. Neither 1 nor 2

Correct Answer : A

Answer Justification :

North Korea shares border with China, South Korea and Russia only. Hence, statement 1 is
correct.

38th parallel popular name given to latitude 38° N that in East Asia roughly
demarcates North Korea and South Korea. Hence, statement 2 is incorrect.

88 Oumuamua, sometime seen in the news, is related to

A. The first confirmed object from another star to visit our solar system

54
Total Marks : 200
Online Prelims TEST - 6 (SUBJECT WISE)
( InsightsIAS Mock Test Series for UPSC Preliminary Exam 2020 )

B. First earth like object found in other solar system


C. Distant comet that may hit earth in the near future.
D. None of the statements (a), (b) and (c) are correct.

Correct Answer : A

Answer Justification :

The first known interstellar object to visit our solar system, 1I/2017 U1 ‘Oumuamua, was
discovered Oct. 19, 2017 by the University of Hawaii’s Pan-STARRS1 telescope, funded by NASA’s
Near-Earth Object Observations (NEOO) Program, which finds and tracks asteroids and comets in
Earth’s neighborhood.

The first confirmed object from another star to visit our solar system, this interstellar
interloper appears to be a rocky, cigar-shaped object with a somewhat reddish hue. The
object, named ‘Oumuamua by its discoverers, is up to one-quarter mile (400 meters) long and
highly-elongated—perhaps 10 times as long as it is wide.

Hence, option (a) is correct.

https://solarsystem.nasa.gov/small-bodies/comets/oumuamua/in-depth/

89 With reference to the GPS-aided Geo augmented navigation (GAGAN), consider the following
statements:
1. It is a joint satellite based augmentation system project between ISRO and Airports Authority of
India.
2. It is inter-operable with other international SBAS systems like US-WAAS and European EGNOS.
3. GAGAN GEO footprint extends from Africa to Australia and has expansion capability for seamless
navigation services across the region.

Which of the statements given above is/are correct?


A. 1 and 2 only
B. 2 and 3 only
C. 3 only
D. 1, 2 and 3

Correct Answer : D

Answer Justification :

The Indian Space Research Organization (ISRO) and Airports Authority of India (AAI) have
implemented the GPS Aided Geo Augmented Navigation-GAGAN project as a Satellite Based
Augmentation System (SBAS) for the Indian Airspace. Hence, statement 1 is correct.

The objective of GAGAN to establish, deploy and certify satellite based augmentation system for
safety-of-life civil aviation applications in India has been successfully completed. The system is

55
Total Marks : 200
Online Prelims TEST - 6 (SUBJECT WISE)
( InsightsIAS Mock Test Series for UPSC Preliminary Exam 2020 )

inter-operable with other international SBAS systems like US-WAAS, European EGNOS,
and Japanese MSAS etc. Hence, statement 2 is correct.

GAGAN GEO footprint extends from Africa to Australia and has expansion capability for
seamless navigation services across the region. GAGAN provides the additional accuracy,
availability, and integrity necessary for all phases of flight, from enroute through approach for all
qualified airports within the GAGAN service volume. Hence, statement 3 is correct.

https://www.isro.gov.in/applications/step-towards-initial-satellite-based-navigation-services-india-ga
gan-irnss

90 Consider the following pairs


National Park State
1.
Tadoba Maharashtra
2.
Mouling Tripura
3.
Rajaji Uttar Pradesh

Which of the pairs given above is/are correct?


A. 1 and 2 only
B. 2 only
C. 1 only
D. 2 and 3 only

Correct Answer : C

Answer Justification :

Tadoba Andhari Tiger Reserve is located in Chandrapur district of Maharashtra state in


India. It is Maharashtra's oldest and largest national park. Created in 1995, the Reserve includes
the Tadoba National Park and the Andhari Wildlife Sanctuary. Hence, pair 1 is correctly
matched.

Mouling National Park is a national park located in the Indian state of Arunachal Pradesh,
spread primarily over the Upper Siang district and parts of the West Siang and East Siang district.
It was the second national park to be created in the state, after Namdapha National Park in 1972.
Hence, pair 2 is incorrectly matched.

Rajaji National Park is an Indian national park and tiger reserve that encompasses the
Shivaliks, near the foothills of the Himalayas. The park is spread over 820 km² and three
districts of Uttarakhand: Haridwar, Dehradun and Pauri Garhwal. Hence, pair 3 is incorrectly
matched.

91 Consider the following statements regarding Low Temperature Thermal Desalination (LTTD)
technology.

56
Total Marks : 200
Online Prelims TEST - 6 (SUBJECT WISE)
( InsightsIAS Mock Test Series for UPSC Preliminary Exam 2020 )

1. It is a process under which the warm surface sea water is flash evaporated at low pressure and the
vapour is condensed with cold deep sea water.
2. LTTD technology does not require any chemical pre and post-treatment of seawater.
3. It is completely indigenous, robust and environment friendly.

Which of the statements given above is/are correct?


A. 1 and 2 only
B. 1 only
C. 2 and 3 only
D. 1, 2 and 3

Correct Answer : D

Answer Justification :

The LTTD is a process under which the warm surface sea water is flash evaporated at low
pressure and the vapour is condensed with cold deep sea water. The LTTD technology does
not require any chemical pre and post-treatment of seawater and thus the pollution
problems are minimal and suitable for island territories. Since no effluent treatment is required, it
gives less operational maintenance problems compared to other desalination processes. Hence,
both statement 1 and statement 2 are correct.

The LTTD technology is completely indigenous, robust and environment friendly. Hence,
statement 3 is correct.

The cost per liter of desalination would depend on the technology used and cost of electricity which
varies from place to place. According to the cost estimates made recently by an independent agency
for LTTD technology, the operational costs per litre of bottled quality fresh water currently works to
be 19 paise.

Since the LTTD technology is not matured for coastal regions of mainland India, except in thermal
power plants located very near to the coast, so far no attempt has been made to introduce such
plants in Andhra Pradesh. The coastal areas would require offshore plant with larger capacity,
which are yet to develop.

http://pib.gov.in/newsite/PrintRelease.aspx?relid=75249

92 Consider the following statements


1. African Union is a continental body consisting of the 55 member states that make up the countries
of the African Continent.
2. Asia continent has most number of countries than any other continent.
3. Tropic of cancer passes Algeria, Niger and Libya.

Which of the statements given above is/are correct?


A. 1 and 2 only
B. 2 only
C. 1 and 3 only

57
Total Marks : 200
Online Prelims TEST - 6 (SUBJECT WISE)
( InsightsIAS Mock Test Series for UPSC Preliminary Exam 2020 )

D. 1, 2 and 3

Correct Answer : C

Answer Justification :

The African Union (AU) is a continental body consisting of the 55 member states that
make up the countries of the African Continent. It was officially launched in 2002 as a
successor to the Organisation of African Unity (OAU, 1963-1999). Hence, statement 1 is correct.

Africa has more countries than any other continent. The continent of Africa consists of 54
sovereign states as well as three dependencies. Hence, statement 2 is incorrect

Tropic of Cancer Passes through:

Algeria

Niger

Libya

Egypt

Saudi Arabia

UAE (Abu Dhabi)

Oman

India

Bangladesh

Myanmar

China

Taiwan

58
Total Marks : 200
Online Prelims TEST - 6 (SUBJECT WISE)
( InsightsIAS Mock Test Series for UPSC Preliminary Exam 2020 )

Mexico

Bahamas

Western Sahara (area claimed by Morocco; Sahrawi Arab Democratic Republic)

Mauritania

Mali

Hence, statement 3 is correct.

93 Consider the following statements regarding National Food Security Act, 2013
1. The Act provides for coverage of upto 75% of the rural population and upto 50% of the urban
population for receiving subsidized foodgrains under Targeted Public Distribution System (TPDS).
2. The eligible persons will be entitled to receive 5 Kgs of foodgrains per person per month at
subsidised prices.
3. The eldest women of the household of age 18 years or above will be the head of the household for
the purpose of issuing ration cards.

Which of the statements given above is/are correct?


A. 1 only
B. 1, 2 and 3
C. 2 and 3 only
D. 1 and 3 only

Correct Answer : B

Answer Justification :

The Act provides for coverage of upto 75% of the rural population and upto 50% of the
urban population for receiving subsidized foodgrains under Targeted Public Distribution
System (TPDS), thus covering about two-thirds of the population. Hence, statement 1 is correct.

The eligible persons will be entitled to receive 5 Kgs of foodgrains per person per month
at subsidised prices of Rs. 3/2/1 per Kg for rice/wheat/coarse grains. The existing Antyodaya
Anna Yojana (AAY) households, which constitute the poorest of the poor, will continue to receive 35
Kgs of foodgrains per household per month. Hence, statement 2 is correct.

Salient features of the Act:

All Antyodaya Anna Yojana (AAY) or the poorest of the poor group, a priority group to receive

59
Total Marks : 200
Online Prelims TEST - 6 (SUBJECT WISE)
( InsightsIAS Mock Test Series for UPSC Preliminary Exam 2020 )

7 kg of subsidised foodgrains per person per month ie. 35 kg of foodgrain/family/month.


General households will be entitled to atleast 3 kg/person/month. Upto 75 percent of the rural
and up to 50 percent of the urban population will be covered by the bill. Of these, at least 46
percent of the rural and 28 percent of the urban population will be designated as priority
households. The rest will be designated as general households.

Pregnant women and lactating mothers will be entitled to meals and maternity benefits of not
less than Rs 6000. It is however restricted to two children only.

The eldest women of the household of age 18 years or above will be the head of the
household for the purpose of issuing ration cards.

Hence, statement 3 is correct.

All beneficiaries will have to pay Rs 3/kg for rice, Rs 2/kg for wheat, Re 1/kg for coarse grains.
These prices can be revised after the first three years, up to the level of the minimum support
price (assured price paid by the Centre to farmers at the time it buys grains from them).

http://vikaspedia.in/social-welfare/social-security/right-to-food

94 Consider the following pairs:


Harvest State/Region Festivals
1. Wangala Mizoram
2. Lohri Punjab
3. Nuakhai Odisha

Which of the above pairs is/are correctly matched?


A. 1 only
B. 2 only
C. 2 and 3 only
D. 1, 2 and 3

Correct Answer : C

Answer Justification :

Lohri is a renowned harvest festival in Punjab that showcases


traditional dance and songs. To kill the chills of winter, the entire family
and neighbours gather around the bonfire and sing together and offers
grains, corns, and nuts to respect and appreciate the grand harvest of
sugarcane crops. Hence, pair 2 is correctly matched.

Wangala is the merriment of 100 drums played by Garo tribes

60
Total Marks : 200
Online Prelims TEST - 6 (SUBJECT WISE)
( InsightsIAS Mock Test Series for UPSC Preliminary Exam 2020 )

of northeast India. This is one of the popular harvest festivals of


India marking the onset of winter. During this festival, Sun God is
worshiped with immense devotion and zeal. Women wear their
traditional colourful clothes and dance during this harvesting festival
while men rhythmically drum their fingers on the traditional drum
pads. Wangala is celebrated in Meghalaya and Assam. Hence, pair 1 is incorrectly matched.

Nuakhai is an age old harvest celebration in Odisha. Locally ‘nua’


means new and ‘khai’ means food. This is not only a popular harvest
festival in India, but also celebrated to appreciate the passing away of
the past and evil days while welcoming the new and beautiful with open
arms. The festival is also known as Nuakhai Parab or Nuakhai
Bhetghat. Hence, pair 3 is correctly matched.

95 With reference to certification marks in India, consider the following statements:


1. Silk Mark is a mandatory certification mark managed by the Silk Mark Organization of India.
2. FPO mark got a mandatory status only after the Food Safety and Standards Act of 2006.
3. India organic is the certification Mark issued by Ministry of Agriculture and Farmers welfare.

Which of the statements given above is/are correct?


A. 1, 2 and 3
B. 1 and 2 only
C. 2 only
D. 2 and 3 only.

Correct Answer : C

Answer Justification :

Silk Mark is a certification mark in India for silk textiles. The mark
certifies that the piece of textile which bears the mark is made of pure
natural silk. The certification is managed by the 'Silk Mark
Organisation of India', a society set up by the state-controlled Central
Silk Board of India. The mark is only advisory in nature and is not
legally endorsed. Hence, statement 1 is incorrect.

The FPO mark is a certification mark mandatory on all processed fruit


products sold in India such as packaged fruit beverages, fruit-jams,
crushes and squashes, pickles, dehydrated fruit products, and fruit
extracts, following the Food Safety and Standards Act of 2006.

The standards have been in force since 1955 by the law of Fruit Products
Order, after which the mark is named, but the mark itself got a
mandatory status only after the Food Safety and Standards Act of 2006. Hence, statement 2 is
correct.

India Organic is a certification mark for organically farmed food products manufactured in

61
Total Marks : 200
Online Prelims TEST - 6 (SUBJECT WISE)
( InsightsIAS Mock Test Series for UPSC Preliminary Exam 2020 )

India. The certification mark certifies that an organic food product conforms to the National
Standards for Organic Products established in 2000.

Those standards ensures that the product or the raw materials used in the product were grown
through organic farming, without the use of chemical fertilizers, pesticides, or induced hormones.
The certification is issued by testing centres accredited by the Agricultural and Processed
Food Products Export Development Authority (APEDA) under the National Program for
Organic Production of the Government of India. Hence, statement 3 is incorrect.

96 Which of the following are the potential benefits of Agro-Forestry in India.


1. Sustainable income for the farmers
2. Conservation of agro-biodiversity
3. Increased productivity of the crops
4. Prevention of soil erosion.

Which of the statements given above is/are correct?


A. 1, 2 and 4 only
B. 1, 2 and 3 only
C. 1, 2 , 3 and 4
D. 2, 3 and 4 only

Correct Answer : C

Answer Justification :

All of them are the potential benefits of Agro-Forestry.

http://vikaspedia.in/agriculture/forestry/agro-forestry

97 Yogyakarta principles, sometime seen in the news, is related to

A. Yoga practices
B. Human Rights
C. Climate Change
D. Sustainable Development

Correct Answer : B

Answer Justification :

The Yogyakarta Principles is a document about human rights in the areas of sexual
orientation and gender identity, published as the outcome of an international meeting of human
rights groups in Yogyakarta, Indonesia, in November 2006.

Hence, option(b) is correct.

62
Total Marks : 200
Online Prelims TEST - 6 (SUBJECT WISE)
( InsightsIAS Mock Test Series for UPSC Preliminary Exam 2020 )

https://yogyakartaprinciples.org/

98 With reference to Chandrayaan-2 mission, consider the following statements:


1. India is the first country in South Asia to launch mission to Moon.
2. India aims to land on and explore the lunar South Pole.

Which of the statements given above is/are correct?


A. 1 only
B. 2 only
C. Both 1 and 2
D. Neither 1 nor 2

Correct Answer : C

Answer Justification :

Chandrayaan-2 mission:

In September 2008, the Chandrayaan-2 mission was approved by the government for a cost of
Rs 425 crore.

It is India’s second mission to the moon.

India is the first country in South Asia to launch mission to Moon. Hence, statement
1 is correct.

It aims to explore the Moon’s South Polar Region. Hence, statement 2 is correct

The mission is an important step in India’s plans for planetary exploration, a program known
as Planetary Science and Exploration (PLANEX).

There are three components of the mission, an orbiter, a lander and a rover.

The mission payloads include — Terrain Mapping Camera which will generate a Digital
Elevation Model (DEM) of the entire moon, Chandrayaan 2 Large Area Soft X-ray
Spectrometer which will test the elemental composition of the Moon’s surface Solar X-Ray
Monitor which will provide solar X-ray spectrum inputs for CLASS.

The orbiter will be deployed at an altitude of 100 kilometers above the surface of the
Moon. The lander will then separate from the orbiter, and execute a soft landing on the
surface of the Moon, unlike the previous mission which crash landed near the lunar south

63
Total Marks : 200
Online Prelims TEST - 6 (SUBJECT WISE)
( InsightsIAS Mock Test Series for UPSC Preliminary Exam 2020 )

pole.

The lander, rover and orbiter will perform mineralogical and elemental studies of the lunar
surface.

The rover is named Pragyan.

The mission’s lander is named Vikram after Dr Vikram A Sarabhai, the Father of the Indian
Space Programme.

https://www.insightsonindia.com/2019/07/23/chandrayaan-2-2/

99 With reference to KUSUM Scheme, consider the following statements:


1. It aims to install grid-connected solar power plants each of capacity up to 2 MW in the rural areas.
2. It is implemented by Ministry of Power

Which of the statements given above is/are correct?


A. 1 only
B. 2 only
C. Both 1 and 2
D. Neither 1 nor 2

Correct Answer : A

Answer Justification :

The Government of India is in the process of formulating a Scheme ‘Kisan Urja Suraksha evam
Utthaan Mahabhiyan (KUSUM)’ which, inter alia, provides for:

installation of grid-connected solar power plants each of capacity up to 2 MW in the


rural areas; Hence, statement 1 is correct.

installation of standalone off-grid solar water pumps to fulfill irrigation needs of farmers not
connected to grid;

solarization of existing grid-connected agriculture pumps to make farmers independent of


grid supply and also enable them to sell surplus solar power generated to DISCOM and get
extra income; and

solarization of tube-wells and lift irrigation projects of Government sector.

64
Total Marks : 200
Online Prelims TEST - 6 (SUBJECT WISE)
( InsightsIAS Mock Test Series for UPSC Preliminary Exam 2020 )

KUSUM scheme is implemented by Ministry of New and Renewable Energy. Hence,


statement 2 is incorrect.

http://pib.gov.in/newsite/PrintRelease.aspx?relid=177489

100 Consider the following statements regarding Van Dhan Scheme


1. The Van Dhan Scheme is an initiative of the Ministry of Tribal Affairs and TRIFED.
2. Scheme is implemented by constituting 10 Self Help Groups of 30 Tribal gatherers.
3. It aims at build upon the traditional knowledge and skill sets of tribals by adding technology and IT
for value addition.

Which of the statements given above is/are correct?


A. 2 only
B. 1 and 2 only
C. 3 only
D. 1, 2 and 3

Correct Answer : D

Answer Justification :

The Van Dhan Scheme is an initiative of the Ministry of Tribal Affairs and TRIFED. It was launched
on 14th April, 2018 and seeks to improve tribal incomes through value addition of tribal products.

The scheme will be implemented through Ministry of Tribal Affairs as Nodal Department at
the Central Level and TRIFED as Nodal Agency at the National Level. At State level, the State
Nodal Agency for MFPs and the District collectors are envisaged to play a pivot role in scheme
implementation at grassroot level. Locally the Kendras are proposed to be managed by a Managing
Committee (an SHG) consisting of representatives of Van Dhan SHGs in the cluster. Hence,
statement 1 is correct.

At unit level, aggregation of produce would be done by SHGs having about 30 members each
forming Van Dhan Vikas ‘Samuh’. The SHGs would also undertake primary value addition of the
MFPs using equipment such as small cutting and sieving tools, decorticator, dryer, packaging tool
etc based on MFPs available in the area. Hence, statement 2 is correct.

It aims at build upon the traditional knowledge and skill sets of tribals by adding technology and IT
for value addition. Hence, statement 3 is correct.

A typical Van Dhan Vikas Samuh would have the following facilities:

Provision for required building/ infrastructure support to be established in one of the


beneficiary’s house/ part of house or Government/ gram panchayat building

Equipment/ Tool Kit comprising of equipment such as small cutting and sieving tools,
decorticator, dryer, packaging tool etc. based on MFPs available in the area

65
Total Marks : 200
Online Prelims TEST - 6 (SUBJECT WISE)
( InsightsIAS Mock Test Series for UPSC Preliminary Exam 2020 )

Fully equipped training facilities for a batch of 30 trainees with provision for raw material for
training purpose and supply of trainee kits (comprising of bag, scribbing pad, pen, brochures,
training manual, booklet etc)

Provisioning of working capital for the SHGs through tie up with financial institutions, banks,
NSTFDC etc

A cluster of ten such SHGs within the same village shall form a Van Dhan Vikas Kendra.
Subject to successful operations of the samuhs in a Kendra, common infrastructure facilities
(pucca Kendra) may be provided to the Kendra in the next phase in terms of building,
warehouse, etc. for use of the samuh members

An illustrative list of major MFPs which may be covered under the initiative are tamarind,
mahua flower, mahua seed, hill broom, chironjee, honey, sal seed, sal leaves, bamboo split,
myrobalan, mango (amchur), aonla (churan/candy), seed lac, tez patta, cardamom, black
pepper, turmeric, dry ginger, cinnamon, coffee, tea, sea buckthorn tea, etc. Apart from these,
any other MFP with potential for value addition may be included.

http://vikaspedia.in/social-welfare/scheduled-tribes-welfare/van-dhan-scheme?content=small

66

Potrebbero piacerti anche